You are on page 1of 58

SAMPLE QUESTIONS

A Collective Arrangement by All Editors and Associate Editors


ANESTHESIA . . . . . . . . . . . . . . . . . . . . . . . . . . . . . 2 NEUROSURGERY . . . . . . . . . . . . . . . . . . . . . . . . . . 26 CARDIAC AND VASCULAR SURGERY . . . . . . . 3 OBSTETRICS . . . . . . . . . . . . . . . . . . . . . . . . . . . . . . 27 CARDIOLOGY . . . . . . . . . . . . . . . . . . . . . . . . . . . . . 4 OPHTHALMOLOGY . . . . . . . . . . . . . . . . . . . . . . . . 28 COMMUNITY HEALTH . . . . . . . . . . . . . . . . . . . . . 5 ORTHOPEDICS . . . . . . . . . . . . . . . . . . . . . . . . . . . . 30 DERMATOLOGY . . . . . . . . . . . . . . . . . . . . . . . . . . . 7 OTOLARYNGOLOGY . . . . . . . . . . . . . . . . . . . . . . . 31 EMERGENCY MEDICINE . . . . . . . . . . . . . . . . . . . 9 PEDIATRICS . . . . . . . . . . . . . . . . . . . . . . . . . . . . . . 33 ENDOCRINOLOGY . . . . . . . . . . . . . . . . . . . . . . . . . 11 PHARMACOLOGY . . . . . . . . . . . . . . . . . . . . . . . . . 38 FAMILY MEDICINE . . . . . . . . . . . . . . . . . . . . . . . . 14 PLASTIC SURGERY . . . . . . . . . . . . . . . . . . . . . . . . 39 GASTROENTEROLOGY . . . . . . . . . . . . . . . . . . . . 16 PSYCHIATRY . . . . . . . . . . . . . . . . . . . . . . . . . . . . . . 40 GERIATRIC MEDICINE . . . . . . . . . . . . . . . . . . . . . 18 RESPIROLOGY . . . . . . . . . . . . . . . . . . . . . . . . . . . . 45 GYNECOLOGY . . . . . . . . . . . . . . . . . . . . . . . . . . . . . 19 RHEUMATOLOGY . . . . . . . . . . . . . . . . . . . . . . . . . 46 HEMATOLOGY . . . . . . . . . . . . . . . . . . . . . . . . . . . . 21 UROLOGY . . . . . . . . . . . . . . . . . . . . . . . . . . . . . . . . . 48 INFECTIOUS DISEASES . . . . . . . . . . . . . . . . . . . . 22 ANSWERS TO SAMPLE QUESTIONS . . . . . . . 49 NEPHROLOGY . . . . . . . . . . . . . . . . . . . . . . . . . . . . 23 KEY FEATURE QUESTIONS . . . . . . . . . . . . . . . . 52 NEUROLOGY . . . . . . . . . . . . . . . . . . . . . . . . . . . . . . 25 KEY FEATURE SCORING GUIDE . . . . . . . . . . . 56

MCCQE 2002 Review Notes

Sample Questions 1

ANESTHESIA
1. Which of the following is the proper positioning for intubation? a) extension of the lower C-spine and extension of the atlanto-occipital joint b) flexion of the lower C-spine and extension of the atlanto-occipital joint c) flexion of the lower C-spine and flexion of the atlanto-occipital joint d) open the mouth only half way to get a good look at the cords e) have the patients head on a high pillow to facilitate intubation 2. Which of the following is NOT part of rapid sequence induction? a) patient breathes 100% O2 for 3-5 minutes prior to induction b) the patient receives bag ventilation immediately following induction c) the Sellick manoeuvre d) pressure placed on the cricroid cartilage e) muscle relaxant given 3. Which of the following methods is sufficient to determine if the EET has passed through the cords? a) chest movement b) absence of abdominal distension c) condensation of water vapour in the EET tube during expiration d) CO2 in exhaled gas as measured by capnogragh e) refilling of reservoir during exhalation 4. Which of the following is CORRECT? a) extracellular fluid volume (ECF) equals 2/3 of total body water b) ECF volume deficit will cause hypotension and bradycardia c) ECF volume expansion will cause increased JVP and S3 d) moderate dehydration means 10% of the ECF volume is lost e) [Na+] determines ECF volume 5. Which of the following is NOT a transfusion reaction? a) fever b) hypercalcemia c) anaphylaxis d) intravascular hemolysis e) extravascular hemolysis 6. Which of the following is NOT a side-effect of opioid analgesics? a) diarrhea b) constipation c) nausea d) biliary spasm e) respiratory depression 7. Opioid analgesics: a) Prevent the release of inflammatory/pain mediators at the site of injury b) Block synaptic transmission along the pain pathway c) Stimulate respiration and increase blood pressure d) Are rarely associated with tolerance e) Cause papillary dilation 8. The most potent inhalation induction agent that is commonly used is: a) enfluane b) isoflurane c) sevoflurane d) nitrous oxide (N2O) e) halothane 9. Contraindications to thiopental include all following EXCEPT: a) cardiac failure b) porphyria c) bowel obstruction d) potential difficult intubation e) status asthmaticus 10. The following is a depolarizing muscle relaxant? a) Rocuronium b) Mivacurium c) Succinylcholine d) Tubocurarine e) Pancuronium 11. Which of the following is FALSE? Epidural analgesia: a) causes uterine relaxation b) controls blood pressure in preeclampsia c) can cause urinary retention d) can contribute to the effect of caval compression e) can cause itching 12. All but one of the following increases the likelihood of regurgitation at the time of induction: a) obesity b) anxiety c) upper airway obstruction d) increased esophageal sphincter pressure e) hiccups 13. Which of the following is used in the treatment of malignant hypertension: a) succinylcholine b) ketamine c) dantrolene d) tubocurarinee e) neostigmine 14. Regarding pre-operative investigations, all of the following are true EXCEPT: a) EKG recommended for those > 40 year old b) pulmonary function test (PFTs) for those with COPD c) Hb and urinanalysis are required for every surgical patient as indicated in the Public Health Act d) hospital policy dictate the indications for tests such as CXR

2 Sample Questions

MCCQE 2002 Review Notes

CARDIAC AND VASCULAR SURGERY


1. Intermittent Claudication is: a) a reproducible discomfort in exercising muscle groups b) secondary to occlusive arterial disease c) a relatively benign condition when treated with risk factor modification and exercise d) may be life-style limiting e) all of the above 2. Ischemic rest pain is: a) unremitting pain in the most distal portion of the affected extremity b) a precursor to gangrene and limb-loss c) associated with pallor on elevation and rubor on dependency d) should be treated with aggressive revascularization surgery e) all of the above 3. An arterial ulcer can be described as: a) red granulating base. Located over a weight bearing area. Non-painful b) red granulating base. Heaped up border with venous engorgement. Located in the gaiter distribution c) whitish, necrotic base. Punched out appearance. No evidence of healing, painful. Located usually on the distal parts of the extremity d) all of the above e) none of the above 4. An aneurysm can be defined as: a) an artery greater than 5 cm in diameter b) a localized enlargement of a vessel greater than 1.5X its expected diameter c) an artery greater than 3 cm in diameter d) a blood vessel greater than 1.5X its expected diameter e) an out-pouching of the aorta 5. The clinical findings of a ruptured abdominal aortic aneurysm include: a) pulsatile abdominal mass b) hypotension c) back pain d) hematemesis e) A, B, and C f) all of the above

MCCQE 2002 Review Notes

Sample Questions 3

CARDIOLOGY
1. All of the following regarding the management of heart failure are true EXCEPT: a) digitalis is indicated in atrial fibrillation resulting in heart failure b) the use of -blockers in acute CHF has been shown to reduce mortality c) calcium channel blockers add no proven survival benefit in patients with CHF d) oral inotropes have a detrimental effect on survival. e) ACE inhibitors improve survival in asymptomatic patients with LVEF < 35% 2. Regarding pericardial disease, which of the following is INCORRECT? a) the ECG changes of acute pericarditis include PR segment depression and ST segment elevation b) autoimmune pericarditis is not associated with acute MI c) a prominent x descent might be expected in the JVP waveform of a patient with a pericardial effusion d) tamponade may result in hypotension, elevated central venous pressure, and pulsus paradoxus e) a relatively small pericardial effusion may result in tamponade if it develops rapidly 3. Regarding syncope: a) the etiology of as many as 50% of cases is never identified b) vasovagal syncope (the common faint) results from a combination of peripheral vasodilatation and relative bradycardia c) a common cause of syncope is a cortical stroke d) the final common pathway of most causes of syncope is usually generalized cerebral hypoperfusion e) cough, micturition, and defecation syncope all result in part from impaired venous return to the heart 4. Which of the following statements is INCORRECT? a) chronic sympathetic nervous system activation causes long-term changes in the myocardium in heart failure b) the development of heart failure requires an initiating myocardial insult c) dilated cardiomyopathy is a common cause of CHF d) poor ventricular compliance results in an S3 e) Cheyne-Stokes breathing is frequently seen in patients with CHF 5. Which of the following statements about valvular cardiac disease is CORRECT: a) pulmonary valve disease is a leading cause of morbidity and mortality b) rheumatic heart diseases is a minor cause of mitral stenosis c) mitral regurgitation leads to LV dilatation d) radiation of a systolic murmur to the clavicle is not commonly seen in aortic stenosis e) mitral valve prolapse leads to a lower JVP 6. Which of the following provides symptomatic relief but has not ben shown to increase longevity in the context of CHF: a) ACE inhibitors b) loop diuretics c) beta-blockers d) spironolactone e) hydralazine and nitrates 7. The least serious contraindication to Thrombolytic Therapy in acute myocardial infarction is: a) cerebral hemorrhage b) active bleeding c) acute pericarditis d) chronic liver disease e) aortic dissection 8. Which of these is NOT a peripheral sign of infective endocarditis: a) Osler's nodes b) splinter hemorrhages c) Janeway lesions d) clubbing e) palmar erythema 9. Which of the following is not one of the major Jones criteria: a) pancarditis b) subcutaneous nodules c) polyarthritis d) Huntingtons chorea e) erythema margination 10. Which of the following is TRUE regarding the JVP: a) 3rd degree heart block is associated with cannon a waves b) a positive Kussmaul sign is not correlated with constrictive pericarditis c) C-V waves are seen in tricuspid stenosis d) atrial fibrillatoin does not affect the JVP waveform e) the y decent occurs during ventricular systole

4 Sample Questions

MCCQE 2002 Review Notes

COMMUNITY HEALTH
1. One rationale for universal hepatitis B vaccination in infants is that: a) the vaccine is not effective in preventing infection in high-risk adults b) no risk factors can be identified in 25% of hepatitis B cases c) hepatitis B infections occur commonly in children d) the HBV vaccine is contraindicated in at-risk pregnant women e) most chronic carriers contract hepatitis B during early childhood 2. Polyvalent pneumococcal vaccine is NOT RECOMMENDED in patients: a) with sickle cell anemia b) with HIV infection c) who take prophylactic penicillin d) under 2 years of age e) who are pregnant or considering pregnancy in the next three months 3. Which one of the following is caused by chronic tobacco smoking? a) increased throat pain during streptococcal infection b) increased ciliary action c) increased anti-protease activity d) hyperplasia of respiratory epithelium e) increased risk of ulcerative colitis and endometrial carcinoma 4. You are director of occupational health for a corporation that has many employees aged over 45 who smoke one or more packs of cigarettes daily and are at increased risk for lung cancer. What strategy for the early detection of lung cancer in asymptomatic individuals would you recommend? a) no strategy has been shown to be effective in reducing mortality b) chest x-ray and sputum cytology every 6 months for high-risk employees c) annual chest x-ray and sputum cytology for high-risk employees d) annual chest x-rayand sputum cytology for all employees e) annual chest x-ray for all employees 5. Regarding obesity: a) exercise, diet and behavioural therapy result in sustained weight loss in 50% of patients b) obesity is the leading cause of preventable death in industrialized countries c) the periodic health exam recommends routine diet counselling and measuring BMI for all individuals d) even a small weight loss is associated with benefit e) switching from soft drinks sweetened with sugar to those with artificial sweetners results in fewer calories consumed and sustained weight loss for most patients 6. Regarding cocaine abuse: a) cocaine abuse is primarily confined to lower social classes, and is often associated with opiate abuse b) cocaine is well absorbed via the oral route c) physical dependence is common, and physical withdrawal symptoms are severe d) cocaine may cause toxic psychosis and hypertension e) cocaine has no current medical indications 7. Pneumonia: a) is the third most common infective cause of death in the developed world b) is very rarely caused by Gram-negative organisms in elderly patients c) is treated at home in less than 50% of patients d) pleuritic pain is a poor prognostic sign e) a poor white cell response is an ominous sign 8. A case control study is designed to analyse a suspected hypothetical association between the development of mania and migraine treatment. This study: a) can derive and estimate of the risk of an individual developing the mania as a consequence of migraine treatment b) will demonstrate that any association found is likely to be causal c) will need controls who are chosen at random from the general population d) will necessitate careful follow-up of a group of patient receiving migraine treatment, and a not treated control group e) will not give biased results if all the patients have been carefully assessed by a senior clinician 9. A double blind trial is planned to compare the utility of glyburide and metformin in the treatment of diabetes mellitus. The main reasons for randomising patients are: a) so that the number of subjects in each group will be identical b) so that the two patient groups will have similar prognostic features c) so that the statistian will not analyse the data in a biased fashion d) so that the investigator does no know in advance what therapy which patient will receive e) to prevent the clinician knowing which drug the patient is taking 10. Which statement is TRUE? a) the assignment of a gender is a nominal variable b) histogram is a useful way of showing how a particular variable is changing in respect to time c) the mean of a sample of positive numbers is invariably greater than the median value d) the mean of a large sample will always increase in size as the size of the sample increase e) approximately 70% of data points are both within one standard deviation of the mean and larger than the mean of a normally distributed data set

MCCQE 2002 Review Notes

Sample Questions 5

COMMUNITY HEALTH

. . . CONT. 15. Fetal infection with Hepatitis B is most likely during the: a) first trimester b) second trimester c) third trimester d) throughout pregnancy e) Hepatitis B is not vertically transmitted 16. Appropriate treatment of a neonate born to a Hepatitis B positive mother is: a) hepatitis b immune globulin (HBIG) at birth b) HBIG at birth and Hepatitis B vaccine at 2, 4, and 6 months of age c) HBIG and Hepatitis B vaccine at birth, 1, 6 months of age d) HBIG at birth and Hepatitis B vaccine at birth, 1 and 6 months of age e) hepatitis B vaccine at 2, 4, and 6 months of age 17. With regards to the vertical transmission of Hepatitis B: a) there is a less than 1% risk if the mother is asymptomatic and HBsAG+ b) can be transmitted through the placenta and through breast milk only c) risk of vertical transmission is greater than 80% if mother is HBsAg+ and HBcAg+ d) chronic active hepatitis B has no effect on prematurity e) hepatitis B vaccine should not be given to a pregnant woman 18. Vertical transmission of HIV can be reduced by giving AZT: a) during pregnancy b) during labour c) at the delivery d) to the neonate e) all of the above

11. Which clinical picture most likely corresponds to a patient who has ingested two bottles of aspirin? a) arterial thrombosis and hypothermia b) hyperpnea, tinnitus, and respiratory alkalosis c) metabolic acidosis, hypokalemia, hyperglycemia d) weakness, ataxia, drowsiness e) facial pallor followed later by hepatic tenderness 12. Who should be screened for lead poisoning in Canada? a) all children with developmental delay b) all children living in housing built before 1960 c) all children living in housing in high-risk districts d) children with unexplained symptom complex of abdominal pain, anorexia, anemia, ataxia, and slurred speech e) none of the above 13. Each of the following patients has a 5-mm reaction to a 5TU PPD skin test. Who should receive therapy for tuberculosis? a) an intravenous drug user known to be HIV-seronegative b) a Vietnamese refugee c) a 30-year-old long-term resident of a mental institution d) a 40-year-old with diabetes mellitus e) a 40-year-old female with known HIV infection 14. A 54-year-old white female who is new to your practice presents with a viral upper respiratory infection. She insists that three days of penicillin is the only thing that ever cures it. You explain your reasons for not using an antibiotic, but she continues to insist. You should: a) treat her symptomatically but do not give antibiotics b) refer her to a physician in your community who has a reputation for frequently using antibiotics c) have the patient sign a medical release before giving her a script for gentamycin d) tell her that if she is not better in three days you will give her an antibiotic as a compromise

6 Sample Questions

MCCQE 2002 Review Notes

DERMATOLOGY
1. All of the following have been implicated in the pathogenesis of acne vulgaris EXCEPT: a) androgen stimulated production of sebum b) Propionibacterium acnes c) consumption of sugars, fats and oils d) hyperkeratinization of follicle lining 2. Which of the following medications is not indicated for the treatment of acne vulgaris? a) benzoyl peroxide b) topical erythromycin c) adapalene d) tazarotene e) metronidazole 3. Which of the following is not a feature of perioral dermatitis? a) papules and inflammatory plaques b) rim of sparing round vermillion border of lips c) predominantly males d) teenagers to mid-adulthood a) treated with topical metronidazole 4. You have been referred a 2 month old with atopic dermatitis. Where are the most likely sites of the dermatitic eruptions in an infant of this age? a) peripheral fingers and toes b) flexural creases of elbows and wrists c) face and extensor surfaces of limbs d) diaper distribution 5. You have been referred an 80 year old man with a history of venous insufficiency, who complains of a constant irritation of his lower legs of 6 months duration. The most likely diagnosis is: a) seborrheic dermatitis b) stasis dermatitis c) atopic dermatitis d) allergic contact dermatitis 6. The cause of cradle cap in infants is most commonly: a) atopic dermatitis b) seborrheic dermatitis c) pityriasis rosea d) alopecia areata 7. The most common causative agents of Impetigo Vulgaris are: a) pseudomonas and streptococcus b) fungi c) GABHS and Staphylococcus aureus d) GABHS and Group B Streptococcus e) Mycoplasmae 8. Staph aureus group II exfoliating toxin is associated with a) bullous impetigo in axillae and groin folds b) psoriatic knee patches c) facial atopic dermatitis d) vitiligo 9. Which of the following is NOT a true difference between erysipelas and cellulitis? Erisypelas Cellulitis a) Upper dermis Lower dermis and subcutaneous fat b) Well demarcated Poorly demarcated c) Group A Strep as Dermatophyte causative agent as causative agent d) commonly on face commonly on extremities 10. Primary syphilis is described by the following EXCEPT: a) excruciatingly painful buttonlike papule b) regional non-tender lymphadenopathy c) VDRL negative initially d) Treated with 2.4 million units benzathine penicillin G given intramuscularly 11. Hand-Foot-and-Mouth Disease is: a) common in the elderly b) caused by staphylococcus aureus c) caused by Coxsackie A16 virus d) chronic e) treated with antibiotic medications 12. Which of the following is not associated with a dermatophytic infection? a) Tinea capitis a) Tinea Corporis a) Condylomata acuminata a) Onychomycosis 13. Which of the following typically presents after acute streptococcal pharyngitis? a) atopic dermatitis b) guttate psoriasis c) pityriasis rosea d) plaque psoriasis 14. Which of the following is associated with gluten intolerance? a) Bullous pemphigoid b) Pemphigus vulgaris c) Dermatitis herpetiformis d) Porphyria cutanea tarda 15. Which is not a feature of lichen planus? a) purple b) polygonal c) pruritic d) purulent discharge e) peripheral distribution 16. All of the following are characteristics of basal cell carcinoma EXCEPT: a) associated with UV radiation b) pearly nodule with telangiectasia c) located on body surfaces not exposed to sunlight d) spread by local invasion e) age of onset >4 0

MCCQE 2002 Review Notes

Sample Questions 7

DERMATOLOGY

. . . CONT. 20. Malignant melanoma involving the papillary dermis is Clark Level: a) I b) II c) III d) IV e) V 21. All of the following are treatments for non-scarring alopecia EXCEPT: a) spironolactone b) minoxidil c) hair transplantation d) intralesional triamcinalone e) finasteride

17. Which is NOT a feature of a cutaneous drug eruption? a) toxic epidermal necrolysis b) angioedema c) phototoxicity d) hypopigmentation e) anaphylaxis 18. Which is NOT a feature of vitiligo and its management? a) associated with thryoid disease and diabetes mellitus b) more common in blacks than whites c) treatment options include PUVA therapy d) destruction of melanocytes e) well-demarcated white macules 19. All of the following are skin diseases associated with diabetes mellitus EXCEPT: a) eruptive xanthomas b) tinea pedis c) acanthosis nigricans d) necrobiosis lipoidica e) pruritis

8 Sample Questions

MCCQE 2002 Review Notes

EMERGENCY MEDICINE
1. A patient presents with a puncture wound to the hand that occurred 12 hours ago. He is not certain if he has been vaccinated for tetanus. Which of the following is the proper management of this patient with respect to tetanus prophylaxis? a) tetanus toxoid b) tetanus immune globulin c) both tetanus toixoid and tetanus immune globulin d) no prohylaxis required 2. A 34 year-old otherwise healthy woman presents in the trauma room after being involved in a motor vehicle collision. She is strongly suspected to have internal bleeding. Her heart rate is 130, blood pressure 85/50, and respiratory rate is 40. Which of the following are likely TRUE? a) she has lost < 1,000 cc of blood b) she will have normal urine output c) fluid replacement should consist of crystalloid and blood d) she has lost > 40% of her blood volume 3. A patient presents to the emergency department. On examination he requires a sternal rub to open his eyes. He is muttering incomprehensively and withdraws his hand when a painful stimulus is applied. His Glascow Coma Scale score is: a) 7 b) 8 c) 9 d) 10 4. With respect to question 3, the most appropriate management of this patients airway is: a) none b) oropharyngeal airway c) nasopharyngeal airway d) endotracheal intubation e) cricothyroidotomy 5. The anticholinergic and sympathomimetic toxidromes shareall of the following features EXCEPT: a) hyperthermia b) maydriasis c) tachycardia d) diaphoresis 6. Which of the following is NOT true regarding the management of asthma patients in the emergency room: a) patients unable to speak with an O2 sat < 90% should be intubated b) peak flow meters are the best way to assess response to therapy c) patient education about the proper use of puffers is an important aspect of treatment d) anti-cholinegics are the first line agents in the treatment of mild asthma e) a silent chest on auscultaton indicates an emergency and requires immediate treatment 7. Which of the following is TRUE regarding hypertensive emergencies? a) there is evidence of end organ damage b) renal failure can be both a cause and an effect of hypertensive emergencies c) the goal of treatment is to lower the blood pressure to normal within 30-60 minutes d) fundoscopic findings may include papilledema and hemorrhages e) all of the above 8. In a head trauma patient the most important feature on history is: a) seizure b) loss of consciousness c) headache d) nausea and vomiting 9. According the Ottawa Ankle Rules, an ankle radiographic series is required in all of the following situation EXCEPT: a) bony tenderness to palpation along the posterior edge of the lateral maleolus b) bony tenderness to palpation along the posterior edge of the medial malleolus c) bruising over either malleolus d) inability to weight bear both immediately following the injury and in the emergency department 10. Which of the following is NOT true with respect to hypothermia? a) chest compressions should be deferred up to 1 minute until it is certain that the patient is pulseless as they can precipitate ventricular tachycardia b) pupils become fixed and dilated at core body temperature below 25C c) ventricular fibrillation becomes a risk at core body temperatures < 30C d) the patient should be rewarmed slowly to avoid an afterdrop 11. Which of the following is FALSE regarding adult patients presenting to the emergency department following a sexual assault? a) they will often present with chief complaints other than sexual assault b) physicians are legally obligated to report the assault to the police even if the victim does not wish it reported c) the patient should have folllow up with an MD at a rape crisis centre within 24 hours d) pregnancy and sexually transmitted disease prophylaxis should be offered if appropriate

MCCQE 2002 Review Notes

Sample Questions 9

EMERGENCY MEDICINE

. . . CONT. 14. Which of the following toxins is NOT a cause of an increased anion gap metabolic acidosis? a) methanol b) ethylene glycol c) salicylates d) iron d) benzodiazepines 15. Which of the following toxins is NOT matched to its specific treatment? a) acetaminophen - N-acetylcysteine b) carbon monoxide - hyperbaric oxygen c) heroin - naloxone d) diazepam - bicarbonate e) methanol - ethanol

12. Which of the following is NOT true regarding allergy and anaphylaxis? a) they are IgE mediated immune response to antigens b) epinephrine is the first line agent for the treatment of minor skin eruptions to topical agents c) signs and symptoms of an allergic reaction vary from minor cutaneous eruptions to cardiovascular collapse, bronchospasm and laryngeal edema d) the ABCs are the first step in the treatment of any allergic reaction 13. Which of the following is FALSE with respect to chest pain? a) all patients presenting with chest pain should receive an ECG b) a normal ECG and normal cardiac enzymes rule an ischemic event c) a careful history is essential in determining the etiology of the chest pain d) reproducible chest pain on palpation of the chest wall does not rule out an acute MI

10 Sample Questions

MCCQE 2002 Review Notes

ENDOCRINOLOGY
1. A patient complains of a non-tender mass over the thyroid region on the left side of her neck. Concerned about a thyroid disorder, you order the appropriate investigations. The results are as follows: TSH: 6.0 Free T4: 20.2 Thyroid antibodies: none RAIU: No hot spots seen The next investigation(s) you choose to do are: a) watch and wait for 3-6 months b) FNA c) surgical biopsy d) trial of L-thyroxine therapy for 6 months e) none of the above 2. An 8 year-old boy is brought to the office because his mother is concerned he is entering puberty already. You examine him and note the beginnings of facial hair, axillary hair and Tanner stage 2 external genitalia. Choose the set of investigations you initially want to do: a) CBC, lytes, testosterone, bone age, CT head b) FSH, LH, testosterone, lytes, bone age, DHEA-S c) FSH, LH, testosterone, cortisol, DHEA-S, 11-OH progesterone, bone age d) lytes, testosterone, DHEA-S, 17-OH progesterone, cortisol, bone age 3. A 2 month-old boy has a Ca of 1.80 (corrected) after an assessment for FTT. Mother informs you she has been consistently breastfeeding without trouble as corroborated by a visiting nurse. What is at the top of your differential? a) malabsorption b) pseudohypoparathyroidism c) mother didnt supplement with DiVisol (Vit D supplement) d) DiGeorge syndrome 4. The triple bolus test of pituitary function works by a rapid succession of IV constituents as follows: a) insulin hypoglycemia mediated rise in GH and ACTH LHRH rise in LH and FSH TRH rise in TSH and PRL b) CRH rise in GH and ACTH LHRH rise in LH and FSH TRH rise in TSH and PRL c) estrogen rise in LH, drop in FSH and PRL insulin rise in GH and ACTH TRH rise in TSH d) cosyntropin rise in GH and ACTH GHRH rise in LH and FSH TRH rise in TSH and PRL 5. A 58 year-old man with a past history of a parathyroidectomy for primary hyperparathyroidism is now in your office complaining of headaches worse in the AM (made worse by a small MVA he credits to a loss of peripheral vision). You plan to: a) send to the Emergency Department for an immediate CT head b) check his calcium to ensure theres no remaining parathyroid tissue c) check for a pheochromocytoma (which you know causes headaches) because you are concerned he has MEN I syndrome d) check for a homonymous hemianopia because you are worried about a pituitary tumour e) check for a bitemporal hemianopia because you are worried about a pituitary tumour 6. Which of the following is not associated with thyroid disease? a) dermatitis herpetiformis b) urticaria c) porphyria cutanea tarda d) vitiligo e) alopecia areata 7. In the treatment of Type I Diabetes, which of the following is TRUE? a) Sulfonylureas are useful as an adjunctive therapy to insulin b) Most patients are adequately controlled with one type of insulin (non-mixed) only c) Once diagnosed with Type I DM, patients must immediately be assessed for retinopathy d) During periods of illness or infection, patients may require additional insulin e) The most common initial presentation is visual disturbance 8. Secondary causes of hyperlipidemia include all of the following EXCEPT: a) obesity b) hyperthyroidism c) diabetes d) nephritic syndrome e) glucocorticoids 9. A 63-year-old woman has had a gradually enlarging goiter for at least 10 years. She has no symptoms of hyperthyroidism. On physical examination, both thyroid lobes are irregular, firm and nontender. Her serum thyroxine (T4) level is 120 nmol/L (normal range is 0.51 - 142 nmol/L) and triiodothyronine (T3) -resin uptake is normal at 30%. The patient is treated with 50 ug of T3 daily in an attempt to decrease the size of the goiter. Several weeks later she develops weakness, palpitations, insomnia, and a tremor; she has lost 2.3 kg (5 lb). The size of the thyroid gland is unchanged; her serum T4 level is now 115 nmol/L. These findings indicate that the patient: a) has developed Graves' disease b) has painless thyroiditis c) has autonomous endogenous thyroid function d) is taking more T3 than was prescribed e) has had a hemorrhage into a thyroid nodule

MCCQE 2002 Review Notes

Sample Questions 11

ENDOCRINOLOGY

. . . CONT. 14. All of the following statements correctly pair a useful medication with its mechanisms of action EXCEPT a) Propylthiouracil (PTU) blocks the coupling reaction in T4 synthesis b) Methimazole (MMU) reduces peripheral conversion of T4 to T3 c) Radioactive iodine destroys follicular cells in the thyroid d) Propranolol blocks the sympathetic components of thyrotoxicosis e) Prednisone may relieve the mechanical exophthalmos and opthalmoplegia by reducing inflammation. 15. All of the following laboratory test results are consistent with the clinical picture EXCEPT: a) decreased T3 resin uptake b) decreased TSH response to a TRH challenge c) decreased serum TSH d) increased serum T4 concentration e) positive test for circulating antibodies against the TSH receptor 16. Which of the following disorders is NOT associated with osteoporosis? a) pheochromocytoma b) prolactinoma c) Cushings syndrome d) Kleinfelters syndrome e) Graves disease. 17. A 41-year-old man has been diagnosed with panhypopituitarism. Which of the following is not part of the appropriate management of this patient? a) L-thyroxine b) hydrocortisone c) fludrocortisone d) testosterone e) Medic Alert bracelet 18. While not always the presenting complaint, the earliest symptom/sign of a pituitary tumour is usually: a) visual disturbance (e.g. bitemporal hemianopsia) b) gonadal insufficiency c) headache d) extraocular muscle paresis e) diabetes insipidus 19. An elderly diabetic woman with mild renal insufficiency presents to the emergency room with confusion. Blood glucose was 1.5 mmol/L. She is treated with glucose IV for 24 hours, discharged and instructed not to take any medications. 6 hours after discharge, she is brought back to the emergency department in a coma. Blood glucose is now 1 mmol/L. Which of the following hypoglycemic agents is the patient most likely using? a) repaglinide b) acarbose c) metformin d) rosiglitazone e) glyburide

10. In deciding upon the ideal dosage of replacement therapy for hypothyroidism, each of the following factors is considered EXCEPT: a) subjective patient response b) findings on physical examination c) response of the radioactive iodine uptake d) plasma TSH level e) level of free thyroid hormone measured in the serum 11. An asymptomatic postmenopausal 54-year-old woman sustained a slight concussion in an automobile accident. X-rays of the skull show an enlarged sella turcica. Computed tomography (CT scan) of the skull shows no density within the sella turcica. This patient most likely has: a) A familial syndrome associated with hypercalcemia and pheochromocytoma b) Pigmentation of skin, buccal mucosa, and pressure areas c) Elevated serum calcitonin levels d) Normal endocrine function e) Hypogonadism with hyposomnia 12. A 32-year-old diabetic did not take his usual dose of 30 units of lente insulin and 5 units of regular insulin because he had an "upset stomach with vomiting". The next day he was admitted to hospital. His plasma glucose was 29.9 mmol/L and the urine was strongly positive for ketones. The serum bicarbonate was 6 mEq/L, arterial pH was 7.05 and serum potassium was 5.4 mEq/L. Which one the following statements is most likely to be TRUE? a) The predominant ketone body in the serum is acetoacetate b) The elevated serum potassium level reflects total body potassium in this patient c) The plasma glucagon level will be increased d) The serum phosphate concentration will rise during insulin therapy e) The patient followed the correct course by withholding insulin in the absence of food intake For 13, 14, and 15. A 32-year-old woman visits her physician because of agitation, weight loss, and inability to sleep. When questioned further, she reveals an increased appetite, and an increased frequency of bowel movements. Previously, she had regular menstrual periods, but now they are less frequent and irregular. During the physical examination, the physician notes that her skin is warm and moist and that she has a fine tremor of the fingers, hyperreflexia, and lid lag. The woman has moderately severe exophthalmos, and her upward gaze seems weak and uncoordinated. 13. Which one of the following disease processes is most likely manifesting itself? a) A thyroid adenoma that is secreting thyroxine b) Inappropriate hypothalamic secretion of TRH c) Graves disease d) Hashimotos disease e) Sick euthyroid syndrome

12 Sample Questions

MCCQE 2002 Review Notes

ENDOCRINOLOGY

. . . CONT. 22. A 35-year-old man gives a history of paroxysmal hypertension, headache, diaphoresis and palpitations. His father has hypertension and hypercalcemia and a paternal aunt died from a thyroid cancer. The physician would be most likely to measure serum levels of: a) calcitonin, calcium, and glucose b) glucagon, insulin, and cholesterol c) thyroid hormone, adrenomedullin, and phosphorus d) CRH, gastrin, and renin

20. A 47-year-old man is admitted to hospital to be evaluated for hypertension and hypokalemia. After the patient has been restricted to a 120 mEq/day Na diet for 1 week, his plasma aldosterone twice the normal level. Which is the next MOST appropriate step? a) fludrocortisone suppression test b) cosyntropin (ACTH) stimulation test c) measurement of plasma renin activity d) CT adrenals e) treat with spironolactone 21. Which of the following is considered diagnostic of diabetes: a) random plasma glucose level of 10 mmol/L b) fasting plasma glucose level of 6.8 mmol/L c) on OGTT, 1 hour plasma glucose level > 11.1 d) on OGTT, 2 hour plasma glucose level > 7.8 e) presence of polyuria and polydipsia and plasma glucose level of 12 mmol/L

MCCQE 2002 Review Notes

Sample Questions 13

FAMILY MEDICINE
1. A 28 year old sexually active woman in a stable relationship for 2 years comes to your office for an annual check-up. Which of the following is NOT a recommended screening tool in the Periodic Health Exam? a) counseling re: sun exposure b) pap smear c) breast self examination d) counseling re: folic acid supplementation e) counseling re: bicycle helmet use 2. A 26 year old woman reports suffering from moderately severe unilateral headaches approximately twice a month for the past year. The headaches are pulsating in quality with associated nausea and photophobia. Physical examination is normal. Which of the following is NOT appropriate abortive treatment for an acute attack? a) NSAIDS b) sumatriptan c) DHE d) amitryptiline e) ergotamine 3. A 63 year old man with a 10 year history of type 2 diabetes is screened for microalbuminuria. Albumin:creatinine ratio is 2.6. Which of the following is the appropriate first step in management? a) repeat albumin:creatinine in one year b) repeat albumin:creatinine test twice over the next 3 months and if abnormal, get 24 hour urine for creatinine clearance c) referral to nephrologist d) adjust dosages of oral hypoglycemic medications e) reduce dietary protein 4. A 38 year old businesswoman and mother of two children presents with a 3 month history of fatigue. Past medical history is remarkable for GDM in her last pregnancy. Pallor noted, otherwise normal physical exam. Which of the following investigations is NOT indicated at this time? a) appropriate assessment for anxiety and depression b) appropriate assessment of current life stressors, past trauma and abuse c) CBC d) serum glucose e) BUN, creatinine 5. The target LDL-C for a very high risk patient (10 year risk CAD > 30%, history of cardiovascular disease, or diabetes) is: a) < 2.5 mmol/L b) < 3.0 mmol/L c) < 3.5 mmol/L d) < 4.0 mmol/L e) < 5.0 mmol/L 6. A 65 year-old newly-diagnosed hypertensive male is about to begin anti-hypertensive medication. Which of the following should you NOT prescribe as first-line therapy: a) angiotensin-converting enzyme inhibitor b) long acting dihydropyridine c) beta-adrenergic antagonist d) low dose thiazide diuretic e) high dose hydrochlorothiazide 7. A 32 year old man presents with a 2 week history of persistent cough and generally feeling unwell. On examination, T= 37.7 , HR= 80, BP = 120/70, RR = 16. Which of the following is INAPPROPRIATE treatment? a) symptomatic antipyretics b) antitussives c) bronchodilator bid d) tetracycline 250 mg qid e) rest and fluids 8. A 40 year old smoker presents with a 3 day history of rhinorrhea, cough, and headache. No vital sign abnormalities present. Which of the following is the best management? a) educate patient about duration of symptoms, suggest hydration, analgesics and dextromethorphan prn b) educate patient about duration of symptoms and have her return to clinic in 2 days c) order CXR d) order sputum culture e) educate patient about smoke exposure and risk of URTI and prescribe erythromycin x 10 days 9. Which of the following is CORRECT regarding depression? a) The lifetime risk of major depressive disorder is 55% for women b) Early treatment of major depressive disorder does not improve outcomes c) Risk of recurrence after 3 episodes of major depressive disorder is 90% d) Most patients will need at least two years of pharmacological treatment e) Most primary care patients with depression should be referred for psychiatric consultation 10. When prescribing the oral contraceptive pill (OCP), which of the following should be done: a) Inform patients that the OCP protects against STDs b) Perform breast, abdominal and pelvic exams and assess blood pressure c) Warn patients that their risk of dysmenorrhea is increased d) Perform a complete physical exam after taking a thorough sexual history e) Perform a Beta-HCG test prior to prescribing OCP

14 Sample Questions

MCCQE 2002 Review Notes

FAMILY MEDICINE

. . . CONT. 13. Acceptable methods of assessing for obesity include the following but: a) BMI b) daily calorie intake c) waist-hip ratio d) percent body fat 14. Which of the following is characteristic of alcohol dependence: a) 23-year-old male college student who consumes an average of 23 drinks per week and exhibits no withdrawal symptoms b) 65-year-old recently retired engineer who binge drinks two times a month c) 43-year-old married female, recently fired, and who has been in 2 MVAs involving alcohol this past year d) 35-year-old male former AA member who has not drunk in 10 years

11. Which of the following statements regarding pharyngitis is/are TRUE: a) The most common etiologic agent is viral b) Bacterial causes for sore throats occur predominately in pre-school aged children c) Delaying treatment for Group A -hemolytic Streptococcal pharyngitis by 48 hours does not increase the risk of rheumatic fever d) a) and c) e) all of the above 12. In the context of low back pain, x-ray imaging should be ordered in these cases except: a) 25-year-old male i.v. drug user with 3 week history of low back pain b) 64-year old female with 48-hour history of back pain and numbness in the right leg extending to the knee c) Previously healthy 37-year-old male with 3 month history of low back pain d) 56-year-old woman with well controlled hypertension and 4 week history of low back pain e) 18-year-old construction worker with 1 week history of low back pain

MCCQE 2002 Review Notes

Sample Questions 15

GASTROENTEROLOGY
1. Pseudomembranous colitis is usually caused by toxins of which organism? a) Staph aureus b) Clostridium difficile c) Clostridium perfringens d) Clostridium botulinum e) Bacteroides fragilis 2. A 13 year old presents with increased serum bilirubin and vague nonspecific complaints. Labs show increase in unconjugated bilirubin with normal liver function tests and negative hepatitis screens. There is no evidence of liver abnormality or hemolysis. The most likely diagnosis is? a) Gilberts syndrome b) biliary atresia c) hepatoma d) cholecystitis e) cholangiocarcinoma 3. After noticing his wife avidly eats clay, starch, and ice, her husband brings her to the ER. History reveals she is a vegetarian, and in her second trimester of pregnancy. The most likely disease afflicting this woman is? a) megaloblastic anemia b) hepatolenticular disease c) gastric carcinoma d) pica e) hypermethioninemia 4. The ecchymosis of the flank seen in patients with acute pancreatitis is named after? a) Trousseau b) Goodpasture c) Grey-Turner d) Cullen e) Wernicke-Korsakoff 5. The best test for a massive lover GI bleed is? a) colonoscopy b) angiography c) radiolabelled RBC scan d) barium enema e) fecal occult blood 6. An elderly woman presents with blood per rectum with a 3 day history of severe ab pain, anemia, hypotension and CAD. The most likely diagnosis is? a) ulcerative colitis b) crohns disease c) irritable bowel disease d) ischemic bowel (colitis) e) infectious diarrhea 7. All are causes of splenomegaly EXCEPT: a) infectious mononucleosis b) thalassemia minor c) congestive heart failure d) sarcoidosis e) diabetes 8. A diagnosis of Irritable bowel sydrome is based on: a) Rome criteria which emphasizes negative features of the disease b) Rome criteria which emphasizes positvie features of the disease c) a diagnosis of not an exclusion d) all of the above 9. Diagnosis of celiac sprue includes all of the following EXCEPT: a) evidence of malabsorption b) abnormal small bowel biopsy c) improvement in sign and symptoms with a gluten- containing diet. d) positive anti-endomysial antibody. 10. You suspect a patient with a fat malabsorption syndrome. Which symptom would you NOT expect? a) night blindness b) metabolic bone disease c) bleeding disorder d) lactose intolerance 11. A 12-year-old boy is jaundiced and has an enlarged liver on palpation. Lab values show increased ALT, increased AST, increased conjugated bilirubin. Kayser-Fleisher rings are noticed on the slit-lamp. A diagnosis of Wilsons disease is made. All of the following are true EXCEPT: a) the primary defect is a mutation in ceruloplasmin b) the patient likely has increased levels of serum free copper and liver copper and increased urinary copper with penicillamine challenge c) Wilsons disease is inherited in an autosomal recessive pattern d) patients can demonstrate increased serum ceruloplasmin e) in normal patients, more than 90% of serum copper circulates bound to ceruloplasmin 12. All of the following can be causes of conjugated hyperbilirubinemia EXCEPT: a) Wilsons disease a) Criggler-Najjar syndrome I a) primary biliary cirrhosis a) gallstones a) viral hepatitis 13. The most common cause of a significant upper GI bleed is: a) Mallory Weiss tear a) angiodysplasia a) peptic ulcer a) dieulafoy lesion a) antritis 14. The medical treatment to be of most benefit in the therapy of both variceal and non-variceal upper GI bleed is: a) octreotide b) omprazole c) ice water lavage d) vasopressin e) ranitidine

16 Sample Questions

MCCQE 2002 Review Notes

GASTROENTEROLOGY

. . . CONT. 16. Which one of the following statements regarding liver failure is TRUE? a) a liver biopsy is a not required with a good history of alcoholism b) encephalopathy may be reversible c) SBP (spontaneous bacterial peritonitis) is usually a result of blunt abdominal trauma d) coagulopathy is a result of poor diet in most patients with cirrhosis e) the size a varix is proportional to the degree of portal hypertension

15. A 50 year old man was admitted for a bleeding duodenal ulcer 2 months ago. He was discharged from hospital with an appropriate course of H. Pylori eradication treatment. He has since completed the therapy and is currently asymptomatic. Which of the following is the BEST non-invasive test to confirm eradition of H. Pylori: a) urea breath test b) endoscopy and histology c) IgG serology d) skin testing

MCCQE 2002 Review Notes

Sample Questions 17

GERIATRIC MEDICINE
1. Which of the following is NOT a cause of urinary incontinence? a) diabetes b) coagulopathies c) restricted mobility d) UTIs e) prostatic disease 2. Which of the following physiological changes affect pharmacokinetics in the elderly? a) increased body albumin b) decreased body fat c) increased GFR d) decreased hepatic blood flow e) increased hepatic phase I reactions 3. Which of the following statements is TRUE: a) The ratio of males to females over 85 in Canada is 2:1 b) Accidents are the most common cause of death in Canadians over 65 c) During normal bereavement, symptoms of depression should last for up to one year d) all of the above e) none of the above 4. The functional assessment of IADLs in the elderly include all of the following EXCEPT: a) food preparation b) ability to climb stairs c) driving d) managing finances e) shopping 5. Which of the following is NOT a cause of failure to thrive in the elderly: a) alopecia b) poor dentition c) depression d) neglect e) dementia

18 Sample Questions

MCCQE 2002 Review Notes

GYNECOLOGY
1. Which of the following is not consistent with PCOD? a) high testosterone b) low LH c) average age of presentation 15-35 years d) low or normal FSH e) hirsutism 2. Which of the following is the most common site of occurrence for endometriosis? a) broad ligament b) uterosacral ligaments c) ovary d) rectosigmoid colon e) appendix f) lung 3. Which of the following are diagnostic characteristics of endometriosis? (Multiple answers) a) constant pelvic pain b) chocolate cysts c) deep dyspareunia d) blueberry spots e) cyclic dysmenorrhea f) infertility g) bladder symptoms h) bowel symptoms i) powder burn lesions 4. Which of the following are considered risk factors for ectopic pregnancy? (Multiple answers) a) alcohol consumption b) previous PID c) use of IUD d) appendectomy e) previous ectopic pregnancy f) use of assisted reproductive techniques g) grand multiparity h) endometriosis 5. Which of the following are considered predisposing factors for candidiasis? (Multiple answers) a) diabetes b) pregnancy c) HIV d) organ transplant recipient e) BCP f) use of a diaphragm g) antibiotic therapy h) hypertention i) cigarette smoking 6. Regarding condylomata/genital warts which of the following is/are TRUE? (Multiple answers) a) spread can be prevented using condoms b) infection can be latent, only detectable with DNA hybridization c) some viral serotypes are associated with an increased incidence of cervical CA d) removal of lesions is proven to decrease recurrence e) lesions present during pregnancy is an absolute indication for C/S f) treatment is through either physical or chemical modalities 7. Which of the following is correct regarding the presentation and treatment of genital herpes? (Multiple answers) a) inguinal lymphadenopathy is present with first infection b) recurrent infections are usually less severe and less frequent c) presentation is four to six months after exposure d) initial presentation is as a painless chancre on vulva e) diagnosis is made on viral culture f) treatment with acyclovir is curative g) outbreak is usually preceded by a tingling, burning prodrome 8. Which of the following are absolutely necessary for the diagnosis of PID? (Multiple answers) a) temperature > 39.0 C b) adnexal tenderness c) cervical motion tenderness d) elevated white count e) positive culture for N. gonorrhea, or C. Trachomatis f) elevated ESR g) lower abdominal pain 9. Regarding PAP smear screening protocol, which of the following are recommended? (Multiple answers) a) should be performed yearly from onset of sexual activity until age 69 b) requires only an endocervical cell sampling c) is equally effective in identifying both SCC and adenocarcinoma d) a patient can be released from screening after two consecutive negative tests e) after three negative tests, screening intervals can be increased up to 3 years f) an inadequate sample requires a repeat PAP within 3 months

MCCQE 2002 Review Notes

Sample Questions 19

GYNECOLOGY

. . . CONT. 13. A 20 year old G0 woman presents to your office with a 4-month history of amenorrhea. She had previously normal and regular cycles since age 12 years. Your initial evaluation of this problem includes: a) Thorough history and physical examination b) A beta-hCG test c) TSH and prolactin levels d) All of the above e) none of the above 14. Absolute contraindications to HRT include: a) undiagnosed vaginal bleeding b) known or suspected cancer of the ovary c) acute renal disease d) obesity e) none of the above 15. Which of the following condition(s) are associated with Polycystic Ovarian Syndrome? a) obesity b) hirsutism c) insulin resistance d) acanthosis nigricans e) All of the above

10. Which of the following are consistent with the clinical presentation of fibroids? (Multiple answers) a) menorrhagia b) abnormal bleeding pattern c) abdominal heaviness d) increased abdominal girth e) amenorrhea f) infertility g) abdominal pain h) difficulty emptying bladder i) difficulty defecating 11. Non-contraceptive benefits of oral contraception pills include: a) Reduced benign breast disease and ovarian cysts b) Reduced dysmenorrhea c) Reduced anemia d) Reduced risk of ovarian carcinoma e) All of the above 12. Regarding infertility: a) It is defined as failure to conceive after one year of regular unprotected intercourse b) It occurs in approximately 50-60% of couples c) Most cases are due to unknown factors d) A normal sperm count from semen analysis is > 5 million sperm/mL e) A hysterosalpingogram plays no role in potential evaluation of infertility

20 Sample Questions

MCCQE 2002 Review Notes

HEMATOLOGY
1. You would expect to find an increased bleeding time in all of the following conditions EXCEPT: a) Osler-Weber-Rendu b) HSP c) ITP d) Celiac disease e) Splenomegaly 2. All of the following characterize chronic ITP EXCEPT: a) onset usually following recent viral infection b) spontaneous remission uncommon c) peak age of onset 20-40 years d) occurs more frequently in women e) insidious onset of bleed 3. Which of the following coagulation factors is NOT effected by chronic liver disease a) factor VII b) factor V c) fibrinogen d) factor VIII e) factor IX 4. What disorder results in prolonged bleeding time and decreased factor VIII coagulation activity? a) Hemophilia A b) Hemophilia B c) von Willebrands disease d) Vitamin K deficiency e) None of the above 5. A young girl with thalassemia is undergoing a blood transfusion and suddenly develops back pain, fever and becomes tachypneic. The likely diagnosis is: a) Delayed hemolytic transfusion reaction b) Febrile non-hemolytic transfusion recaction c) Acute hemolytic transfusion reaction d) Circulatory overload e) Graft vs. host disease 6. Which of the following is not accurate for iron stores: a) Serum Fe b) Transferrin c) Ferritin d) Hemosiderin e) TIBC 7. The most common cause of cold antibodies other than idiopathic is: a) Drug induced b) Routine detection c) Secondary to infection d) Secondary to lymphoproliferative disorder e) Autoimmune disease 8. Which of the following blood smear results are most likely to indicate G6PD deficiency? a) Rouleaux b) Howell-Jolly bodies c) Basophilic stipling d) Heinz bodies e) Tear drop cells 9. All of the following may be seen in aplastic anemia EXCEPT: a) Anemia b) Thrombocytopenia c) Leukopenia d) Splenomegaly e) increase frequency of infections 10. The cause of beta thalasemia minor is: a) Increase HbA2 production b) Reduced B chain production c) Increased HbF production d) No B chain production e) Increased B chain production 11. Metabolic disturbances that may be seen in acute myelogenous leukemia include: a) high uric acid, high calcium, high magnesium b) high uric acid, low calcium, low magnesium c) high uric acid, high calcium, high phosphate d) high uric acid, low calcium, low phosphate e) low uric acid, high calcium, high phosphate 12. Clinical features of chronic myelogenous leukemia include all EXCEPT: a) left upper quadrant pain and fullness b) pruritus c) peptic ulcers d) priapism e) jaundice 13. The MOST COMMON presentation of essential thrombocythemia is: a) weight loss, fever b) bleeding c) thrombosis d) asymptomatic e) splenomegaly 14. A 22-year old male student presents with enlarged cervical lymph nodes that become painful after alcohol consumption. The only remarkable feature on history is an 8 lb weight loss over 2 months. The most likely diagnosis would be: a) ALL b) Hodgkins lymphoma c) Non-Hodgkins lymphoma d) CLL e) multiple myeloma 15. Possible treatments for multiple myeloma or its complications include all EXCEPT: a) corticosteroids b) bisphosphonates c) radiation therapy for bone lesions d) renal transplant e) plasmapheresis

MCCQE 2002 Review Notes

Sample Questions 21

INFECTIOUS DISEASES
1. Group A Strep is least likely to cause which of the following complications: a) scarlet fever b) necrotizing faciitis c) impetigo d) subacute bacterial endocarditis e) glomerulonephritis 2. Septra is used in AIDS patients to prevent which opportunistic organism? a) Pneumocystis carinii b) M. tuberculosis c) S. pneumoniae d) CMV e) Cryptococcus 3. Severe bloody, afebrile diarrhea is associated with what bacterial infection? a) Salmonella infection b) Enteroinvasive E.coli infection c) Enterohemorrhagic E. coli infection d) Enterotoxigenic E. coli infection e) Giardiasis 4. Human Herpes Virus 6 is associated with what disease? a) Infectious mononucleosis b) CMV infection c) Herpes Simplex infection d) Roseola e) Chicken Pox 5. Cats scratch disease is caused by what organism? a) Leishmania spp. b) Bartonella henselae c) Treponema pallidum d) Toxoplasmosa gondii e) Ancylostoma braziliensei 6. Predictors of HIV progression include the following EXCEPT: a) CD4 cell count b) plasma HIV RNA levels at set point c) onset of HIV related symptoms d) age at initial infection e) method of transmission (i.e. anal intercourse versus vaginal intercourse) 7. Protozoa are associated with all of the following except a) unicellular organism b) produce larvae c) do not cause eosinophilia d) indefinite lifespan e) mulitply within host 8. Inwhich disease would Donovan bodies be present? a) genital warts b) lymphogranuloma venereum c) syphilis d) granuloma inguinale e) chancroid 9. Which disease is NOT caused by Chlamydia sp.? a) lymphogranuloma venereum b) pelvic inflammatory disease c) granuloma inguinale d) inclusion conjunctivitis e) nongonococcal urethritis 10. If a patient comes to your office complaining of chest pain and a cough producing a current jelly-like sputum, which organism would be the most likely cause of his/her pneumonia? a) Streptococcus pneumoniae b) Haemophilus influenzae c) Pseudomons aeruginosa d) Influenza virus e) Klebsiella pneumoniae 11. Which organ in the human body is the most sensitive to disseminated intravascular coagulation in a case of meningitis? a) thyroid b) adrenal c) lateral ventricle of the brain d) spleen e) aorta 12. The Negri body is a pathogonomiic sign of: a) acute bacterial meningitis b) arbovirus encephalitis c) rabies d) polio e) chronic meningitis 13. If untreated, this infection can lead to cholangiocarcinoma. a) Schistosomiasis b) Clonorchis sinesis c) Strongyloidosis d) Trypanosoma e) E. coli 14. A 10 year old girl from El Salvador is brought to the pediatric outpatient clinic. On the growth charts, she is < 1 percentile. A CBC reveals a profound anemia. She has moderate edema of the lower extremities and face. Stool is sent for ova and parasites (O&P), and the lab reports the presence of helminth eggs. The most likely parasite compatible with her clinical presentation is? a) hookworm b) Strongyloidosis c) pinworm d) pork tapeworm 15. A 48 year old make who received a bone marrow transplant 6 months previously for chronic myelogenous leukemia is admitted with wedge-shaped infiltrates on chest x-ray. He is experiencing severe respiratory difficulty and is coughing up blood. Despite aggressive treatment, he dies shortly after admission. At autopsy, pulmonary infarcts are found. The pulmonary vessels contain fungal hyphae with acute angle branching. The most likely diagnosis in infection with: a) Asperigillus spp. b) Candida spp. c) Blastomycosis d) Pneumocystis carinii

22 Sample Questions

MCCQE 2002 Review Notes

NEPHROLOGY
1. A 63 year old woman with a long history of chronic renal failure is seen because of fatigue. She gets tired after walking 4 blocks, but is comfortable at rest. Her only medication is a calcium channel blocker for hypertension. There is no history suggestive of bleeding. Her physical examination reveals pallor but no other abnormalities. Lab investigations reveal that a Hgb = 85 g/L, MCV = 88 fL, ferritin = 210 ug/L, vitamin B12 = 210 pmol/L, a normal RBC folate, and serum creatinine of 379 umol/L. Two months ago, her Hgb level was 90 g/L. The MOST APPROPRIATE THERAPY for her anemia AT THIS POINT is: a) administration of erythropoietin b) dialysis c) renal transplantation d) vitamin B12 injections e) oral iron 2. A 22 year old woman with a history of type 1 diabetes mellitus is brought to the emergency room in coma. Blood tests indicate a wide anion gap metabolic acidosis. If her metabolic acidosis is due solely to diabetic ketoacidosis, which of the following findings is NOT consistent: a) blood glucose increased b) serum ketones increased c) serum osmolar gap increased d) extracellular fluid volume reduced e) urine dip positive for ketones 3. A patient in acute renal failure comes to the emergency room. His serum potassium value is 8.0 mM. Of the following ECG changes listed below, which ONE is NOT consistent with hyperkalemia? a) wide QRS complexes b) peaked T waves c) peaked P waves d) ventricular fibrillation e) prolonged PR interval 4. A chronic dialysis patient who passes no urine has missed 3 hemodialysis treatments in the last week and comes into the emergency room with a serum potassium value of 8.5 mmol/L. An ECG shows cardiac changes consistent with severe hyperkalemia. Blood glucose is 4.5 mmol/L. Which ONE of the following would be the best action to take: a) b) c) d) e) Give the patient a prescription for an oral potassium exchange resin and arrange dialysis within 24 hours Give insulin and glucose intravenously Give insulin and calcium gluconate intravenously and inhaled salbutamol Give insulin, calcium gluconate and glucose intravenously and arrange for urgent dialysis Give inhaled salbutamol and larger doses of intravenous furosemide 5. Which of the following is the LEAST APPROPRIATE indication for dialysis in the setting of acute renal failure? a) extracellular fluid volume overload unresponsive to diuretics b) hyperkalemia unresponsive to medical attempts to lower the plasma potassium level c) plasma creatinine greater than 300 umol/L in an otherwise asymptomatic patient d) uremic pericarditis e) uremic encephalopathy with seizures 6. The COMMONEST cause of acute renal failure in hospitalized patients is: a) acute crescentic glomerulonephritis b) acute tubular necrosis (ATN) c) cyclosporine nephropathy d) HIV nephropathy e) obstructive uropathy 7. The COMMONEST cause of death in dialysis and renal transplant patients is: a) cardiovascular disease b) HIV-associated complications from repeated infusions of blood products c) infection d) neoplasia e) suicide 8. A 21 year old woman is referred for edema of the ankles that developed 3 weeks earlier. Physical examination shows moderate bilateral ankle edema. The chest was clear and heart sounds were normal. Jugular venous pressure was 1 cm above the sternal angle. Urinalysis shows 3+ protein and occasional red and white blood cells. Plasma creatinine was normal and plasma albumin was reduced at 36 g/L. Which of the following is the LEAST likely renal diagnosis? a) acute tubular necrosis (ATN) b) membranous glomerulonephritis c) minimal change glomerulonephritis d) lupus nephritis e) post-infectious glomerulonephritis 9. A 66 year old man is referred to you for hypertension which was not noted until this year, despite yearly physical examinations for many years. Apart from recent headaches he has no complaints and was not taking any medications. Blood pressure in your office was 175/100 mm Hg with a pulse of 80/min. Examination of the optic fundi showed arteriolar narrowing. Bilateral femoral bruits were present. Urinalysis showed 1+ protein and granular casts. Plasma creatinine was 210 umol/L. The MOST LIKELY diagnosis is: a) atheromatous emboli b) bilateral renal artery stenosis c) essential hypertension d) pheochromocytoma e) proliferative glomerulonephritis

MCCQE 2002 Review Notes

Sample Questions 23

NEPHROLOGY

. . . CONT. 13. Treatment of hypertension caused by bilateral renal artery stenosis should NOT include which of the following? a) weight reduction if obese b) drug therapy with angiotensin-converting enzyme inhibitors c) percutaneous transluminal renal angioplasty d) surgical renal revascularization e) sodium restriction 14. Hyperkalemia is reduced by each of the following EXCEPT: a) administration of calcium gluconate intravenously b) administration of glucose, insulin and sodium bicarbonate intravenously c) oral administration of potassium-exchange resins with sorbitol d) hemodialysis with low potassium dialysate 15. The anemia of chronic renal failure is usually due to which of the following? a) malabsorption of iron b) vitamin B12 deficiency c) blood loss d) decreased erythropoietin production e) associated inflammatory conditions

10. A 36 year old man presents with ankle edema and an elevated jugular venous pressure. Plasma creatinine is 350 umol/L. Past history reveals a polyarthritis for 6 months treated with ibuprofen and allergic rhinitis. Physical exam reveals several active joints in both hands and an itchy skin rash over the trunk. BP = 170/100 mm Hg and he has moderate peripheral edema. Urinalysis shows a trace of protein, trace blood and on microscopy many white blood cells and white blood cell casts are seen. The MOST LIKELY diagnosis is: a) atheromatous emboli b) interstitial nephritis c) lupus nephritis d) membranous glomerulonephritis e) renal artery stenosis 11. Acute diffuse proliferative glomerulonephritis is usually accompanied by each of the following findings EXCEPT: a) red blood cell casts on microscopic urinalysis b) proteinuria c) pigmented casts on microscopic urinalysis d) blood clots in the urine e) none of the above 12. The anion gap is increased in metabolic acidosis associated with each of the following EXCEPT: a) diabetic ketoacidosis b) renal tubular acidosis c) acute tubular necrosis d) ethylene glycol intoxication e) lactic acidosis

24 Sample Questions

MCCQE 2002 Review Notes

NEUROLOGY
1. 32 year old woman with right-hand pain waking her up at night. Physical exam reveals mild weakness of right thumb adduction and some thenar wasting. What is the most likely diagnosis? a) cervical radiculopathy b) carpal tunnel syndrome c) tendinits d) syringomyelia 2. 35 year old woman with one week history of bilateral leg weakness. Physical exam reveals moderate leg weakness, mild arm weakness, and decreased reflexes. What is the most likely diagnosis? a) Lyme disease b) Parkinson's disease c) Multiple Sclerosis (MS) d) Guillian-Barr syndrome 3. Patient with mild left-sided hearing loss and absent left corneal reflex. Where is the lesion? a) left cerebellar pontine angle b) lateral medulla c) left CN V d) left cochlea 4. 75 year old man with sudden onset of severe leg weakness. Physical exam reveals bilateral leg weakness, loss of pain and temperature sensation with intact vibration and positon sense in both legs. What is the likely diagnosis? a) anterior cerebral artery territory stroke b) lumbar radiculopathy c) anterior spinal artery occlusion d) diabetic polyneuropathy 5. Which of the following is NOT consistent with amyotrophic lateral sclerosis (ALS)? a) distal arm and leg weakness b) coexistence of both upper and motor neuron signs c) slowed motor nerve conduction velocities d) fasciculations 6. 52 year old woman with incontinence and dizzy spells. Physical exam reveals rigidity, bradykinesia, postural instability, mild ataxia, and postural hypotension. What is the likely diagnosis? a) vitamin B12 deficiency b) Parkinsons disease c) normal pressure hydrocephalus d) multiple systems atrophy 7. Patient with dizziness, left arm and leg weakness, loss of pain and temperature senasation in the left face and right extremeties. What is the likely diagnosis? a) lateral medullary syndrome b) migraine with aura c) middle cerebellar artery territory stroke d) benign paroxysmal positional vertigo (BPPV) 8. Choose the ONE CORRECT statement. A lesion of the left cerebellum could cause? a) akinesia b) right arm and leg ataxia c) left arm and leg weakness d) spasticity of the right arm and leg e) dysmetria on finger to nose testing of the left arm 9. The following statements concerning the pathophysiology of Parkinsons disease are correct, EXCEPT: a) symptoms appear after there has been a substantial loss of the pigmented neurons of the substantia nigra b) the internal segment of the globus is the major outflow center of the basal ganglia c) the globus pallidus interna is normally excitatory to the thalamus d) a pallidotomy, a surgical lesion placed in the globus pallidus is helpful in some patients with Parkinsons disease e) dopaminergic neurons of the substantia nigra project to the putamen 10. Choose the ONE CORRECT statement regarding Wernickes aphasia: a) patients with Wernickes aphasia have dysprosodic speech b) weakness of the face and arm are commonly found in patients with Wernickes aphasia c) the speech of patients with Wernickes aphasia is fluent with normal or slightly increased speed d) patients with Wernickes aphasia almost universally fail to understand the commance take off your glasses e) patients with Wernickes aphasia cannot write but they are still able to read 11. All of the following statements regarding aphasia are correct, EXCEPT: a) the left hemisphere is dominant for language even in many left handed patients b) patients with Brocas aphasia often have upper motor weakness of the right arm and face c) patients with Brocas Conduction, and Wernickes aphasias all have difficulty with repetition d) both Wernickes and Brocas areas are supplied by the middle cerebral artery e) patients with transcortical type aphasias have a lesion which damages both Wernickes and Brocas cortical areas 12. A patient who is unable to use a comb but has no muscle weakness is likely to have a lesion in which part of the central nervous system? a) the hand area of the motor cortex b) the vermis of the cerebellum c) Brocas area d) part of the premotor or prefrontal cortex e) posterior parietal cortex

MCCQE 2002 Review Notes

Sample Questions 25

NEUROSURGERY
1. Which of the following is FALSE regarding intracranial dynamics? a) normal intracranial pressure is 6-15 mmHg (8-18 cm H2O) b) the relationship between an expanding intracranial mass and the resultant rise in intracranial pressure is linear c) cerebral blood flow depends on cerebral perfusion pressure and cerebral vascular resistance d) lumbar puncture is contraindicated in patients with known or suspected intracranial mass lesions 2. Which of the following is NOT a classic finding in someone with raised intracranial pressure? a) headache with nausea and vomiting b) respiratory changes c) increased blood pressure d) tachycardia 3. An obsese 30 year-old woman presents with headache and nausea, and a bilateral decrease in visual acuity. She has no recent history of trauma and is otherwise well, but takes tetracycline for acne. CT and MRI scans of her head with and without contrast are normal. Of the following options, what is the most likely diagnosis? a) normal pressure hydrocephalus b) benign intracranial hypertension c) meningioma d) acute subdural hematoma 4. What is the most common type of primary brain tumour in adults? a) astrocytoma b) medulloblastoma c) meningioma d) vestibular schwannoma 5. Which of the following is NOT a characteristic CT feature of a brain abscess? a) perilesional hypodensity b) central hypodensity c) homogenous contrast enhancement d) ring enhancement with contrast 6. A 45 year-old man presents with headache. On examination, you notice that he has a large chin and brow, as well as large clammy hands. MRI of his head reveals a large midline suprasellar mass. What visual deficit might you expect in this gentleman? a) right monocular blindness b) amaurosis fugax c) left homonymous hemiaopia d) bitemporal hemianopia 7. What is the most appropriat initial investigation in the diagnosis of subarachnoid hemorrhage? a) lumbar puncture b) CT without contrast c) MRI d) CT with contrast 8. Which of the following is NOT part of the management of vasopasm following subarachnoid hemorrhage? a) maintaining blood pressure at or below 120/80 b) nimodipine c) IV solutions to decrease hematocrit levels d) angioplasty 9. A 70 year-old man complains of trouble walking. On examination, you find that he has left leg weakness and hyperreflexia, but relatively normal power and reflexes in his right leg. Furthermore he has decreased position and vibration sense in his left leg relative to his right, but decreased pin prick sensation in his right leg relative to his left. MRI of his lumbar spine reveals a tumour compressing the mans spinal cord. Where is the tumour? a) lateral to the right side of the lumbar spinal cord b) lateral to the left side of the lumbar spinal cord c) within the centre of the lumbar spinal cord d) anterior and central to the lumbar spinal cord 10. A 29 year-old bodybuilder presents complaining of right arm pain and weakness. Examination reveals a decreased triceps reflex on the right and decreased sensation on the right middle finger. MRI shows a right posterolateral intervertebral disc hernation. Where is the herniation ? a) C4-5 interspace b) C5-6 interspace c) C6-7 interspace d) C7-T1 interspace 11. What is the most common cause of an epidural hematoma? a) ruptured middle cerebral atery b) ruptured anterior cerebral artery c) ruptured posterior communicating artery d) ruptured middle meningeal arter 12. What is the typical appearance of an acute subdural hematoma on noncontrast CT? a) diffuse intraparenchymal hypodense mass b) hyperdense biconvex mass c) hypodense intraventricular mass d) hyperdense concave mass 13. Which of the following is NOT a classic finding in someone with carpal tunnel syndrome? a) positive Tinels syndrome b) hand pain sometimes awakening patient at night c) hypothenar muscle wasting d) positive Phalens sign 14. What is the most common location for pediatric brain tumours? a) anterior cranial fossa b) posterior cranial fossa c) middle cranial fossa d) sella turcica 15. What is the term for the group of hindbrain abnormalities involving some degree of cerebellar hernation or hypoplasia? a) Chiari malformation b) Dandy-Walker malformation c) craniosynostosis d) myelomeningocele

26 Sample Questions

MCCQE 2002 Review Notes

OBSTETRICS
1. Polyhydramnios is associated with all of these EXCEPT: a) premature labour b) cord prolapse c) postpartum hemorrhage d) Potters syndrome e) trisomy 18 2. With regard to thromboembolism and pregnancy all of the following are true EXCEPT: a) Risk increases with maternal age b) Warfarin is teratogenic c) There is an increase in factors I, VII, VIII, IX, X, XII d) Heparin crosses the placenta e) The uterus compresses veins contributing the venous stasis 3. Which of the following are TRUE with regard to normal labour: a) Labour commences with onset of regular painful contractions in the presence of a dilated cervix b) Retraction of the head during the second stage suggests an undiagnosed malposition c) The average time for the second stage is one hour in the multiparous woman d) The second stage includes both a latent and active stage e) Signs of placental separation include a gush of bright red placental blood 4. Which of the following is associated with profuse painless vaginal bleeding in the third trimester? a) abruptio placenta b) umbilical cord prolapse c) degenerating fibroid d) placental insufficiency e) placenta previa 5. Which of the following is NOT a prerequisite to the use of forceps in obstetrical delivery? a) patient must be fully dilated b) full bladder c) adequate anaesthesia d) position of fetus known e) ruptured membranes 6. The most common cause of postpartum hemorrhage is: a) retained placenta b) uterine atony c) vaginal lacerations d) cervical lacerations e) uterine fibroids 7. Which of the following is NOT associated with severe preeclampsia? a) elevated liver transaminases b) thrombocytopenia c) blood pressure greater than 160/110 d) oliguria e) hematemesis 8. A 28 year old G1P0 woman at 35 weeks presents to the labour floor with painless vaginal bleeding. Which of the following should NOT be done? a) vaginal examination b) complete blood count c) crossmatch blood d) ultrasound e) fetal heart rate monitor 9. Which of the following is TRUE regarding malpresentation of a fetus? a) It is associated with an increased risk of congenital anomaly b) Compound presentation precludes a vaginal delivery c) It commonly occurs in association with an anthropoid pelvis d) It may be corrected during labour by intravenous oxytocic agents e) It should be treated by immediate stabilizing induction if the membranes rupture 10. All of the following are true about the renal system in pregnancy EXCEPT: a) The GFR is 60% greater than normal by 12 weeks b) There is a link between UTI and low birth weight c) Acute glomerulonephritis is a rare condition and is usually diagnosed as preeclampsia d) Asymptomatic bacteriuria has a frequency of 5% e) The creatinine and urea decrease in the normal woman compared with non-pregnant values 11. The diagnosis of pregnancy is related to which of the following? a) May be delayed as there is commonly a small amount of blood loss at the time of the first missed period b) Nausea and vomiting usually precede amenorrhea c) Portable doppler machine may detect a fetal heart at 6 weeks d) In the absence of biophysical and biochemical diagnostic aids a bimanual examination to detect Hegars sign should be performed e) The beta subunit of HCG is similar to corresponding units on LH,FSH and TSH giving false positive results on immunosorbent assays 12. All of the following are prerequisites for labour suppression (tocolysis) EXCEPT: a) intact membranes b) live fetus c) absence of fetal distress d) no dilatation e) necessary personnel if tocolysis fails 13. With regards to fetal heart monitoring, which of the following is TRUE: a) Early decelerations are related to a vagal response to head compression b) An acceleration is defined as an increase of at least 20 bpm lasting at least 20 seconds c) Normal range is 100-180 bpm d) A variable deceleration is the least common change seen during labour e) A late deceleration peaks at the same time as the uterine contraction 14. Which of the following is NOT true: a) With rubella infection, the greatest risk to the fetus occurs in the first trimester b) Genital herpes lesions is an indication for a caesarean section c) AZT decreases the incidence of vertical transmission of HIV d) Varicella vaccine is safe during pregnancy e) All women should be screened for Hepatitis B 15. Which of the following is TRUE: a) The puerperium refers to the first 2 weeks after delivery b) The uterus should reach the non-pregnant state with in 1-2 weeks c) Foul smelling lochia suggests endometritis d) Lochia changes in time from lochia rubra to lochia serosa to lochia alba e) Postpartum blues are rare

MCCQE 2002 Review Notes

Sample Questions 27

OPHTHALMOLOGY
1. Which of the following is NOT a contraindication to pupil dilation: a) narrow anterior chamber b) iritis c) neurologic abnormality requiring pupillary evaluation d) iris supported anterior chamber lens implant 2. In myopics, the eyeball is: a) too long b) too short c) non-spherical d) crooked 3. The most common cause of exophthalmos in children is: a) hyperthyroidism b) orbital cellulitis c) orbital tumours d) orbital hemorrhage 4. Eversion of the lower lid margins is termed: a) trichiasis b) entropion c) ectropion d) chalazion 10. Severe photophobia is most characteristic of: a) acute glaucoma b) cataracts c) iritis d) corneal abrasion 11. Which is NOT a management step in iritis: a) pressure patch b) pupil dilation c) systemic analgesics d) medical work-up to determine etiology 12. Cataracts can be due to all of the following but: a) aging b) diabetes mellitus c) increased lipids d) uveitis 13. Which of the following require the most urgent attention: a) central retinal vein occlusion b) cataracts c) scleritis d) central retinal artery occlusion 14. Management for primary angle closure galucoma includes all of the following except: a) laser iridotomy b) mydriatic drops c) topical beta-blockers d) IV hypertonic mannitol e) systemic carbonic anhydrase inhibitors 15. Which of the following is NOT associated with primary angle closure glaucoma: a) steroid use b) nausea and vomiting c) hyperopia d) painful red eye e) opacified cornea 16. Which of the following is NOT a cause of a relative afferent pupillary defect: a) multiple sclerosis b) optic neuritis c) dense cataract d) large retinal detachment e) central retinal vein occlusion 17. Which of the following is associated with impaired pupil dilation: a) sympathetic stimulation b) Adies tonic pupil c) parasympathetic understimulation d) Horners syndrome 18. The most common ocular infection in HIV is: a) herples simplex b) pnuemocystis carinni c) cytomegalovirus d) candida e) toxoplasmosis

5. The most common type of lid carcinoma is: a) adenocarcinoma b) squamous cell carcinoma c) sebaceous cell carcinoma d) basal cell carcinoma 6. Which conjunctivitis typically begins as unilateral and progresses to the opposite eye: a) bacterial b) viral c) allergic d) chlamydial 7. Which of the following is associated with contact lens wear: a) vernal conjunctivitis b) iritis c) scleromalacia perforans d) giant papillary conjunctivitis 8. Which of the following should never be given for corneal abrasions: a) topical analgesics b) topical antibiotics c) pressure patch d) topical cycloplegics 9. Dendritic lesions are characteristic of: a) herpes zoster keratitis b) arcus senilus c) herpes simplex keratitis d) scleritis

28 Sample Questions

MCCQE 2002 Review Notes

OPHTHALMOLOGY

. . . CONT. 22. Which of the following tests for a phoria: a) Hirschberg test b) cover test c) cover-uncover test d) holler test e) accomadation reflex 23. The following are true in the management of chemical burns to the eye EXCEPT: a) alkali burns have a worse prognosis than acid burns b) an alkali burn should be neutralized immediately c) an IV drip with water should be set-up for irrigation in the emergency room d) cyclopegic drops should be administered 24. In differentiating the causes of red eye, photphobia is most characteristic of: a) cataract b) conjunctivitis c) angle closure keratitis d) acute glaucoma e) acute iritis

19. In a newly diagnosed patient with type 2 diabetes, appropriate screening for retinopathy should involve: a) begin screening five years after diagnosis b) repeat in four years and thereafter annually c) begin screening three years after diagnosis, then repeat annually d) begin screening after age 50; earlier if poor glycemic control 20. Non-proliferative changes in diabetic retinopathy include all of the following EXCEPT: a) microaneurysms b) retinal edema c) intraretinal microvascular anomalies d) retinal edema e) dot and blot hemorrhages 21. Grade 5 involvement in graves disease refers to: a) corneal involvement b) soft tissue involvement c) sight loss due to optic neuropathy d) proptosis

MCCQE 2002 Review Notes

Sample Questions 29

ORTHOPEDICS
1. A 35 year old male manual labourer sustained a displaced subcapital hip fracture after falling at work. The fracture was reduced and fixed with 3 cannulated screws. Five months after the operation, he presents to your clinic with worsening hip pain. What is the most likely diagnosis? a) Nonunion b) Osteonecrosis c) Loosening of the cannulated screws d) Malunion e) Osteomyelitis 2. A 25 year old professional basket ball player lands on his right foot while it is in a pronated and externally rotated position. He hears a snap and is unable to weight bear on his right foot. An ankle X-ray reveals significant talar shift without a visible fibular fracture. The most appropriate next step is: a) Closed reduction of the ankle mortise followed by application of a cast b) Bedrest for 4 weeks followed by intense physiotherapy c) X-ray the right knee to rule out a Maisonneuve fracture d) Open reduction with internal fixation of the right ankle e) Splint ankle and encourage weight bearing 3. A 75 year old lady slips on a throw rug in her living room and falls. An X-ray reveals a displaced subcapital hip fracture. Prior to her fall, the patient lived alone, per formed all ADLs independently and enjoyed golfing. The most appropriate management of this fracture is: a) Moores unipolar hemiarthroplasty b) Bedrest for 6 weeks c) Reduction with internal fixation using 3 cannulated screws d) Bipolar hemiarthroplasty e) Total hip replacement 4. A 21 year old presents to your office after injuring her knee in a soccer game. She states that the knee clicks when she walks and has locked on several occasions. On exam there is an effusion and the knee is grossly stable. The most likely diagnosis is: a) Anterior cruciate ligament tear b) Meniscal tear c) Osteoarthritis d) Bursitis e) Medial collateral ligament tear 5. All of the following regarding Achilles tendon rupture are true, EXCEPT: a) Positive Thompsons test b) Palpable gap over Achilles tendon c) Weak plantar flexion d) May occur secondary to steroid injection e) Treat by casting foot in dorsiflexion

30 Sample Questions

MCCQE 2002 Review Notes

OTOLARYNGOLOGY
1. Regarding laryngeal cancer, which of the following is FALSE? a) hoarseness appears early b) involved nodes are not palpable in 35% of cases c) distant metastasis appears early d) direct extension is common e) it is 90% five-year curable when limited to one cord 2. All of the following are removed in radical neck dissection EXCEPT: a) sternocleidomastoid muscle b) external carotid artery c) internal jugular vein d) spinal accessory nerve e) submaxillary gland 3. Which of the following factors is NOT associated with squamous cell carcinoma of the larynx? a) male sex b) age in fifth and sixth decades c) history of woodworking d) large ethanol intake e) tobacco smoking 4. In LeFort I fractures, the fragment consists of all of the following EXCEPT: a) upper teeth and palate b) lower portions of the pterygoid processes c) portions of the walls of both maxillary antra d) nasal spine e) bridge of the nose 5. In general, traumatic perforations of the tympanic membrane: a) are a surgical emergency b) will heal spontaneously in most cases c) usually require operative repair d) require microsurgical repair e) require a graft for repair 6. The MOST common organism in acute otitis media of older children and adults is: a) Staphylococcus b) Streptococcus c) Hemophilus influenzae d) Klebsiella pneumoniae e) Pseudomonas 7. A 65-year-old white male who has been smoking pipes since early adulthood notes a small patch of white on the lateral anterior portion of the tongue. The patch is not painful for the first month, but gradually becomes more painful as it begins to enlarge and ulcerate. The MOST likely diagnosis is? a) benign nonspecific ulceration b) leukoplakia (benign) c) epulis d) carcinoma of the tongue e) ranula of the tongue 8. The diagnosis in question 7 may be confirmed by: a) a positive Wasserman test b) a positive lupus erythematosus (LE) preparation c) a biopsy of the lesion d) diagnostic mandibular and maxillary x-rays e) observation of further progression of the disease 9. A cholesteatoma is: a) an atherosclerotic lesion b) a dermal collection of cholesterol salts c) epithelial debris in the middle ear d) a yellow papule beneath the oral tongue e) retained cerumen 10. Small, malignant tumours of the larynx that are intrinsic in origin and have not spread beyond the larynx are BEST treated by: a) irradiation b) laryngofissure c) total laryngectomy d) total laryngectomy and radical neck dissection e) radium needle implants 11. Clinical features of facial fractures frequently include all of the following EXCEPT: a) deformity b) facial nerve paralysis c) anesthesia over areas of trigeminal branch distribution d) ocular disparity e) malocclusion of the teeth 12. The MOST sensitive test for nasal fracture is: a) history b) physical diagnosis c) plain x-ray studies d) magnetic resonance imaging e) computed tomography (CT) scanning 13. Mixed tumours of the salivary gland: a) are most common in the submaxillary gland b) are usually malignant c) are most common in the parotid gland d) usually cause facial paralysis e) are associated with calculi 14. In epistaxis, what percentage of the cases will respond to ten minutes of direct pressure? a) 10% b) 30% c) 70% d) 90% e) 0% 15. Which of the following is NOT a cause for conductive hearing loss? a) otitis media b) otosclerosis c) noise-induced hearing loss d) perforation of the tympanic membrane e) ossicular chain disruption

MCCQE 2002 Review Notes

Sample Questions 31

OTOLARYNGOLOGY

. . . CONT. 23. What is the MOST common cause of acquired subglottic stenosis? a) motor vehicle trauma b) prolonged endotracheal intubation c) chronic bronchitis d) tracheoesophageal fistula e) previous tracheal surgery 24. A 5-year-old child has persistent serous effusions in both ears for 6 months after a routine acute infection. He has a 40-dB condutive heraring loss in both ears and has been having trouble in school. What would be the BEST treatment for this child? a) observe the child for another 3 months b) prescribe amoxicillin for 10 days c) recommend hearing aids d) place ventilating tubes e) prescribe prophylactic antibiotics for 3 months 25. A 3-year-old child has has eight episodes of acute otitis media in 6 months and has difficulty resolving the effusions between infections. What should be done to effectively eliminate the infections? a) continuing treating each infection as it arises b) place ventilating tubes c) prescribe prophylactic antibiotics for 6 months d) remove the tonsils e) give IV antibiotics for 4 weeks after infectious disease consultation 26. The following clinical entities are common causes for tinnitus EXCEPT: a) high-frequency hearing loss b) Mnires disease c) ototoxic drugs d) loud noise exposure e) acute otitis media 27. Vertigo is very common in all of the following conditions EXCEPT: a) vestibular neuritis b) Mnires disease c) presbycusis d) viral labyrinthitis e) benign paroxysmal positional vertigo 28. The fastest, safest means of establishing a surgical airway is: a) endoscopic intubation b) tracheotomy under local anesthesia c) tracheotomy under general anesthesia d) cricothyrotomy e) puncture through the thyroid membrane

16. Conductive hearing losses are usually reversible. Which of the following conditions is reversible by surgical treatment? a) otosclerosis b) presbycusis c) sudden hearing loss d) ototoxicity e) meningitis 17. What is the BEST treatment for most cases of sensorineural hearing loss associated with aging (presbycusis)? a) nothing b) hearing aid c) ear trumpet d) diuretic therapy e) labyrinthectomy 18. The MOST common benign lesion of the external ear is: a) melanoma b) chondrodermatitis nodularis chronicus helicus c) cerumenoma d) actinic keratosis e) exostosis of the canal 19. MOST of the infectious and/or inflammatory diseases involving the middle ear space are secondary to: a) ciliary dyskinesia b) resistant pathogens c) eutstachian tube dysfunction d) tobacco abuse e) allergic diathesis 20. Acute otitis is: a) a rare condition b) the most common reason ill children visit the doctor c) usually not accompanied by pain and fever d) caused by coliform bacteria e) treated by placing ventilating tubes 21. All of the following ar indications for tonsillectomy EXCEPT: a) six to seven episodes of tonsillitis in 1 year b) airway obstruction secondary to tonsillar hypertrophy c) repeat ear and sinus infections d) recurrent peritonsillar abscess e) very large asymmetric tonsil in an adult 22. The MOST common cause for infant stridor, accounting or 60% of the cases, is: a) subglottic hemangioma b) vocal cord paralysis c) laryngomalacia d) congenital webs e) laryngeal cleft

32 Sample Questions

MCCQE 2002 Review Notes

PEDIATRICS
1. A 6 month-old infant presents in the winter with fever, cough, wheezing, tachypnea and decreased appetite. A chest radiograph shows hyperaeration and streaky perihilar infiltrates bilaterally. You diagnose bronchopneumonia. Which organism would most likely be causing this child's infection? a) Chlamydia pneumoniae b) Mycoplasma pneumoniae c) Streptococcus pneumoniae d) Haemophilus influenzae e) respiratory syncytial virus 6. Which of the following investigations is most helpful in the assessment of a child presenting with an acute asthma attack, who responds poorly to treatment? a) white cell count and differential b) arterial blood gases c) chest x-ray d) pulmonary function tests e) sweat chloride test A 6 year-old boy is brought to emergency with a 5 day history of fever, cough, and poor appetite, but no vomiting. On exam, he appears unwell, is febrile, and has crusty nasal discharge and a wet-sounding cough. Chest x-ray reveals a pulmonary infiltrate in the right middle lobe. He is admitted to hospital for pneumonia. What is the best management for this child? a) observe for 24 hours with IV fluids only b) observe for 24 hours with IV fluids and acetaminophen c) give oral amoxicillin and acetaminophen d) give IV ampicillin and oral acetaminophen e) give oral erythromycin and acetaminophen In comparing breast milk and formula, which of the following statements is NOT correct: a) breast milk has a higher percentage of protein b) breast milk has whey:casein ratio of 60:40 c) breast milk contains leukocytes, complement and lysozymes d) breast milk has a lower concentration of iron than iron fortified formulae e) breast milk has an optimal calcium:phosphorus ratio of 2:1 Most umbilical hernias in children: a) need strapping b) resolve spontaneously c) require elective surgery d) require immediate surgery e) are associated with a higher incidence of inguinal hernias

7.

The following case pertains to questions 2 and 3: A 12 month-old girl is brought to the emergency department for the second time in 2 days for vomiting and passage of 8 to 10 watery stools per day. 2. Of the following, which provides the best estimate of the patients volume deficit: a) weight change since the beginning of the illness b) hydration of mucous membranes, skin turgor, and level of consciousness c) pulse, blood pressure, and peripheral capillary filling time d) serum electrolytes e) serum urea nitrogen and creatinine levels The patient has lost 0.6 kg. She is moderately lethargic and has dry mucous membranes and reduced skin turgor. Blood pressure is 80/40 mm Hg, and pulse is 120 per minute; capillary refill is reasonably brisk. Lab studies reveal: sodium 131, potassium 4.8, chloride 101, bicarbonate 16 mEq/L, urea nitrogen 24 mg/dL and creatinine 0.6 mg/dL. The best strategy for managing this child is to: a) hospitalize for administration of IV fluid therapy b) administer an oral rehydrating solution while the child is under medical supervision for 4-6 hours c) instruct the parents about oral rehydration at home d) hospitalize after giving 20 mL/kg of 0.9% saline IV e) instruct parents on use of soy formula Which of the following cases is most suggestive of child abuse? a) a child who clings to her parent but shies away from the emergency physician. b) a parent who refuses to leave his/her child alone with the physician, despite the physicians repeated requests. c) a parent who claims that his child broke her arm after falling off her bike. d) a child with recurrent urinary tract infections despite antibiotic prophylaxis. e) a child with old-looking bruises on both elbows and shins.

8.

3.

9.

4.

10. A newborn male spits up his first feeding and develops bilious emesis with subsequent feedings. On physical exam he appears ill, has a scaphoid abdomen and absent bowel sounds. Abdominal x-ray shows air in the proximal small bowel, but a paucity of air in the distal digestive tract. The most likely cause for this infant's vomiting and clinical finding is: a) antral web b) choledochal cyst c) Hirschsprung disease d) tracheoesophageal fistula e) volvulus 11. Breastfeeding is contraindicated if the mother has: a) acute EBV infection b) acute HepA infection c) chronic HepB infection d) asymptomatic HIV infection e) none of the above

5) A 14 year-old girl has refused to go to school 3 times in the past 4 months. She says, "My tummy hurts," but she cannot point to where it bothers her. Her appetite is good, her bowel movements are normal, and she is sleeping well at night. She is a healthy-looking girl with no abnormal findings on physical exam. What is the most likely diagnosis? a) benign abdominal mass compressing her duodenum b) diaphragmatic hernia c) functional abdominal pain d) gastroesophageal reflux disease e) early peptic ulcer disease

MCCQE 2002 Review Notes

Sample Questions 33

PEDIATRICS

. . . CONT. 19. Which of the following statements is true? a) cancer is the second most common cause of death in children b) Hodgkin's lymphoma is the most common childhood cancer c) Hyperploidy in leukemic cells is a poor prognostic indicator d) Wilm's tumor is rarely associated with other congenital abnormalities e) neuroblastomas usually occur in late adolescence 20. A 5 year-old girl with hypogammaglobulinemia and absent immunoglobulin A (IgA) receives infusions of immune globulin monthly. Shortly after her most recent infusion began, she developed hypotension, wheezing, and several urticarial lesions. Which of the following best explains her reaction? a) anaphylactic reaction due to IgE anti-IgA antibodies b) gram-negative sepsis due to contaminated immune globulin c) idiosyncratic reaction due to rapid infusion of immune globulin d) serum sickness reaction from foreign serum in the immune globulin e) transfusion reaction due to ABO incompatibility 21. When prophylactic antibiotic therapy is used for tooth extraction in a patient with a ventricular septal defect: a) therapy is started 24 hours prior to the procedure b) a throat swab should be taken prior to the procedure c) a second generation cephalosporin (cefuroxime) is the therapy of choice d) therapy is given for one week following the extraction e) erythromycin is the drug of choice for those allergic to penicillin 22. Which of the following statements about stuttering in a 4 year-old is incorrect: a) it is characterized by intermittent difficulty in producing a smooth flow of speech b) it is more than 3 times more frequent in girls than in boys c) it is exacerbated by anxiety d) more than 30% of children who stutter recover spontaneously e) none of the above 23. A woman who is positive for hepatitis B surface antigen (HBsAg), but negative for hepatitis B antigen (HBeAg), delivers at term. What would be the best management for this woman's infant? a) administer gamma globulin intramuscularly immediately and at 1 month of age b) administer hepatitis B (HB) vaccine immediately and at 1 month and 6 months of age c) administer hepatitis B immune globulin (HBIG) if cord blood is positive for HBsAg d) administer HBIG and HB vaccine immediately, and HB vaccine again at 1 month and 6 months of age e) advise mother that breastfeeding is contraindicated 24. Regarding sexual abuse of children, each of the following statements is true EXCEPT: a) no genital injury is found in the majority of patients b) father-daughter incest is more common than brother-sister incest c) most assailants are unknown to the victim d) laboratory findings usually show no presence of sperm e) half of the abused children come from single parent families

12. A 10 month-old child develops low-grade fever and sunburn-like erythema over much of the body, but most prominently in the intertriginous areas. Within 36 hours, sheet-like desquamation is noted in the flexures and around the mouth. The mucous membranes are spared. Which of the following is the most likely diagnosis: a) Kawasaki disease b) staphylococcal scalded skin syndrome c) Stevens-Johnson syndrome d) toxic epidermal necrolysis e) toxic shock syndrome 13. Which of the following is a contraindication to breast feeding in Canada: a) allergic disease in the family b) infantile diarrhea c) marijuana smoking d) HIV infection e) maternal use of ibuprofen 14. Central cyanosis in the newborn infant is most often caused by: a) congenital heart disease b) lung disease c) central nervous system disease d) methemoglobinemia e) hypoglycemia 15. In a premature infant who is suspected of having necrotizing enterocolitis (NEC), each of the following is correct EXCEPT: a) septicemia is associated with an increased risk of NEC b) NEC is thought to be caused by systemic hypertension c) the finding of air in the portal vein indicates severe illness d) respiratory distress increases the risk of NEC e) Apgar scores inversely correlate with the risk of NEC 16. Which of the following is the most significant risk factor for the development of childhood asthma: a) family social background b) parental asthma c) stress in the family d) parental smoking e) presence of pets in the house 17. Which of the following statements about sickle cell disease is true? a) all patients with sickle cell disease have a homozygous HbSS genotype b) sickle cell disease causes a severe chronic anemia that is not routinely transfusion dependent c) patients have increased susceptibility to infection by nonencapsulated organisms d) patients usually present with sickle cell crises within one month of age e) splenic dysfunction usually does not occur until the child enters his/her teens 18. Which of the is more characteristic of platelet abnormalities and not coagulation defects? a) hematomas b) hemarthrosis c) petechiae d) minimal bleeding from small cuts e) positive family history

34 Sample Questions

MCCQE 2002 Review Notes

PEDIATRICS

. . . CONT. 32. A 16 year-old male has a 4 day illness of abrupt onset consisting of fever, sore throat, rhinorrhea, cough and mild abdominal pain. He has a red tonsillopharyngeal area, a whitish exudate on the tonsils and tender anterior cervical nodes. The most likely cause of his infection is: a) adenovirus b) Group A beta-hemolytic Streptococcus c) Epstein-Barr Virus d) Neisseria gonorrhea e) Staphylococcus 33. A 13 month-old infant boy has chronic diarrhea, poor appetite, irritability and growth failure. He had been well previously, developing until diarrhea began at 3 months of age. Findings include weight loss less than 5th percentile and length at 25th percentile, cachectic appearance, wasted extremities and protuberant abdomen. His labs are: albumin 23, protein 40. Stool positive for reducing sugars and negative for enteric pathogens and ova and parasites. These findings are most consistent with a) Celiac disease b) cow milk allergy c) Crohns disease d) cystic fibrosis e) none of the above 34. Which of the following is most likely to provide the basis for making the correct diagnosis in an infant or child who has failure to thrive (FTT)? a) blood chemistries b) cultures c) history and physical d) radiograph studies e) none of the above 35. A 8 year-old boy has had paroxysmal abdominal pain since his parents separated 6 months ago. Which of the following symptoms would support an organic basis for his disease? a) headaches accompanies the pain b) pain located in the periumbilical region c) pain awakens child at night d) symptoms last < 1hr e) none of the above 36. The most common cause of chronic diarrhea in a 6 month - 36 month old child is: a) chronic non-specific diarrhea b) disaccharidase deficiency c) enteric infection d) malabsorption e) protein intolerance 37. Which of the following is NOT required for a diagnosis of juvenile rheumatoid arthritis? a) arthritis in at least one joint b) arthritis lasting for at least 6 weeks c) positive rheumatoid factor d) onset before the age of 16 e) other causes of arthritis excluded

25. The viral infection most likely to cause CNS involvement and focal neurological findings is : a) coxsackievirus b) herpes simplex c) enterovirus d) rabies e) rhinovirus 26. Which of the following predisposes infants to chronic otitis media? a) bottle-feeding in upright position b) abnormal tympanic membrane formation c) environmental factors such as daycare and passive smoking d) allergies e) none of the above 27. All infants less than 3 months of age who have fever and no localizing signs should have all of the following evaluations EXCEPT: a) blood cultures b) careful history and physical exam c) chest x-ray d) CBC (with differential) e) urine culture 28. All the following statements regarding transient tachypnea of the newborn (TTN) is true EXCEPT: a) infants born by C-section are at increased risk for developing TTN b) residual pulmonary function disability is common among infants who have TTN c) the incidence of TTN is higher than Respiratory Distress Syndrome (RDS) among term infants d) TTN shows marked improvement with 12-24 hrs 29. Antibiotic prophylaxis against infective endocarditis is required for all of the following EXCEPT: a) rheumatic valve lesions b) prosthetic heart valves c) isolated secundum atrial septal defect d) pacemaker leads e) patent ductus arteriosus 30. Causes of microcytic anemia include all of the following EXCEPT: a) excessive cow's milk intake b) iron deficiency c) folic acid deficiency d) thalassemias e) lead poisoning 31. The latest time after the onset of group A hemolytic Streptococcus pharyngitis that initiation of penicillin therapy can be expected to prevent acute rheumatic fever is: a) 24 hrs b) 48 hrs c) 96 hrs d) 9 days e) 2 weeks

MCCQE 2002 Review Notes

Sample Questions 35

PEDIATRICS

. . . CONT. 45. All of the following are appropriate in the initial assessment of a child with failure to thrive EXCEPT: a) diet history b) social history c) measurement of height, weight and head circumference d) growth hormone levels e) bone age x-ray 46. Which of the following statements about a child with short stature is CORRECT? a) the bone age is delayed in a child with constitutional growth delay b) height crosses a major percentile line on the childs growth chart c) weight is more affected than height in a child with an endocrine deficiency d) karyotyping part of the routine investigation of all children with short stature e) growth hormone replacement is helpful in a child with familial growth delay 47. Which of the following developmental milestones in not a cause for concern? a) 15 month old not walking b) 12 month old not talking c) 12 month old who does not search for hidden objects d) 3 year old unable to stand on one foot momentarily e) 6 month old with a persistent grasp reflex 48. All of the following immunizations should be administered to a 7 year-old child who is a recent immigrant, with unknown vaccination status EXCEPT: a) tetanus b) MMR c) diptheria d) pertussis e) polio 49. A 3 year old girl has a 2 month history of left knee swelling and morning stiffness. There is no history of fever or rash. The child appears very healthy. The most likely tentative diagnosis is: a) pauciarticular juvenile rheumatoid arthritis (JRA) b) systemic lupus erythematosus (SLE) c) rheumatic fever d) Kawasakis disease e) Henoch-Schonlein purpura 50. An 8 year old boy has had 2 episodes of loss of awareness. These last 5 seconds and consist of eyes and head turning to one side. EEG reveals 3Hz spike and wave activity. First line therapy would include: a) ACTH b) phenytoin c) carbamazepine d) ethosuximide e) phenobarbital

38. A child presents with bilateral shin pain. Which of the following suggests that this is NOT growing pains? a) pain is poorly localized b) pain awakens the child at night c) no fever or rash d) pain abates with reassurance and massage e) child may limp in the morning from stiffness 39. Which of the following steps is NOT indicated in the management of croup? a) keeping the child calm b) hydration c) antipyretics d) antibiotics e) humidified oxygen 40. Which of the following pairs shows the CORRECT stage of normal development? a) 6 months - pulls to stand b) 12 months - pincer grasp c) 18 months - handedness d) 2 years - draws a cross e) 3 years - tells a story 41. Which of the following is NOT characteristic of a functional murmur? a) pansystolic murmur b) murmur varies with position c) variably split S2 d) murmur becomes louder with fever e) no extra clicks 42. Which of the following is the most common congenital heart lesion in children? a) atrial septal defect b) ventricular septal defect c) patent ductus arteriosus d) Tetralogy of Fallot e) coarctation of the aorta 43. Choose the INCORRECT statement about breastfeeding: a) for healthy term babies, breastfeeding is recommended over formula feeding b) breastmilk contains more amino acids than cows milk. c) exclusively breastfed babies should be supplemented with iron after 6 months of age d) assessment of adequate intake can be done by counting the number of wet diapers in a day e) an HIV-positive mother should not breastfeed 44. All of the following are characteristics of a child with Pervasive Developmental Disorder (PDD) EXCEPT: a) abnormal speech pattern b) stereotypic behaviours, such as head-banging or hand-flapping c) abnormal cognitive function d) consuming interest in one topic or activity e) tendency to reciprocate in peer interactions

36 Sample Questions

MCCQE 2002 Review Notes

PEDIATRICS

. . . CONT. 55. A 1100 gram infant born at 36 weeks gestation develops respiratory distress syndrom and requires supplemental oxygen until 4 months of age. A CXR then shows hyperinflation of both lungs and cystic changes in the bases. The most likely diagnosis is: a) bronchopulmonary dysplasia b) cor pulmonale c) presistant pulmonary hypertension d) pneumonia e) cystic adenomatoid malformation of the lung 56. Breat feeding is a contraindication to which of the following vaccinations? a) diptheria b) influenza c) MMR d) hepatitis B e) none of the above 57. A 6 month old infant presents in the winter with fever, cough, wheezing, tachypnea and decreased appetite. A CXR shows hyperaeration and streaky perihilar infiltrates bilaterally. You diag nose bronchopneumonia. Which organism whould likely be causing this childs infection? a) chlamydia pneumonia b) mycoplasma pneumonia c) streptococcus pneumonia d) haemophilus influenza e) respiratory syncitial virus 58. Prostaglandin E1 infusion can be used for the initial management in each of the following neonatal heart lesions EXCEPT: a) pulmonary atresia b) coarctation of the aorta c) Tetralogy of Fallot d) patent ductus arteriosus e) hypoplastic left heart syndrome

51. Turner syndrome is associated with the following clinical features EXCEPT: a) major learning difficulties b) short stature c) primary amenorrhea d) broad chest e) wide carrying angle at elbows 52. Which statement concerning delayed puberty is false? a) defined absence of pubertal development by age 13 in girls and age 14 in boys b) the most common cause is constitutional delay c) chronic disease may cause growth delay d) delayed puberty in males is more suggestive of pathology than delayed puberty in females e) bone age is an important investigation in determining the cause of delayed puberty in both males and females 53. A 3 year old Asian girl presents with a fever of 6 days, red lips and tongue, bilateral nonpurulent conjunctivitis, cervical lymphadenopathy and a polymorphic rash. The most likely diagnosis is: a) Streptococcal pharyngitis b) Kawasakis disease c) infectious mononucleosis d) adenoviral infection e) none of the above 54. Which of the followng statements concerning minimal lesion glomerulonephritis is correct? a) it is a common cause of nephrotic syndrome in children, but it is a very rare cause of nephrotic syndrome in adulthood b) glomerular filtration rate is usually preserved c) it is usually unresponsive to treatment such as prednisone and cyclophosphamide d) it is most often accompanied by severe hypertension e) the urine typically shows red blood cell casts (suggestive of proliferative glomerulonephritis)

MCCQE 2002 Review Notes

Sample Questions 37

PHARMACOLOGY
1. The first pass effect refers to: a) the first time the drug reaches its site of action b) passage of the drug from the skin into the body c) metabolism of the drug by the liver d) passage of the drug from the lung into the systemic circulation e) metabolism of the drug by kidneys 2. A patient with hypoalbuminemia requires: a) a higher dose of a highly protein bound drug b) the same dose of drug protein binding doesnt matter c) a lower dose of a highly protein bound drug d) no drug at all its just too dangerous 3. Compared to water soluble drugs lipid soluble drugs: a) are larger molecules b) have higher ionizations c) diffuse more easily through membranes d) more easily absorbed if given IV 4. The primary site of absorption of most drugs is the: a) mouth b) skin c) stomach d) small intestine e) lungs 5. The primary site of biotransformation of most drugs is the: a) liver b) lung c) small intestine d) skin e) kidneys 6. The main route of excretion for most drugs is the: a) liver b) lung c) small intestine d) skin e) kidneys 7. Regarding phase I drug reactions which is NOT correct: a) they are oxidation, reduction or hydrolysis reactions b) they involve conjugation with polar endogenous substrates c) they increase the water solubility of the drug d) they include the P450 mediated reactions 8. Regarding first order kinetics for drug metabolism which is NOT true: a) a constant fraction of drug is eliminated per unit of time b) elimination is based on drug concentration c) drugs may go from zero to first order kinetics when the receptor is saturated d) it is a non-linear relation 9. A drug with a half life of 24 hours will take ____ to reach steady state: a) 12 hours b) 24 hours c) 48 hours d) 96 hours e) 120 hours 10. Regarding the figure below 100%
Response e.g. % of receptors blocked

60% 50% 25%

B Potency A>B>C Efficacy A=C>B Log Dose

a) b) c) d) e)

Drug A is less potent than drug B Drug A is less efficacious than drug B Drug A is more efficacious than drug C Drug A is equally efficacious to drug C Drug A is equally potent to drug C

11. A drug with a narrow therapeutic index: a) never requires therapeutic drug monitoring b) always require therapeutic drug monitoring c) can be given over a wide dose range with little worry about toxicity d) has a TD50 at a lower concentration than the ED50 e) has an ED50 that is very close to the TD50 12. The following is TRUE regarding type A adverse drug reactions a) they account for 80% of all adverse drug reactions b) they are an extension of the drugs pharmacological effect c) they are predictable d) they require discontinuation of the drug e) they require a dose reduction of the drug

38 Sample Questions

MCCQE 2002 Review Notes

PLASTIC SURGERY
1. A 23 year-old male was involved in a snowmobile accident and sustained a closed displaced humeral shaft fracture. His neurovascular status is normal. You realign the fracture and place him in a splint. Prior to discharge he states he has numbness over the dorsum of this hand and he is unable to extend his metacarpophalangeal joints. Which of the following would you suspect to be the cause? a) ulnar nerve was damaged at the time of the fracture b) radial nerve was damaged at the time of the fracture c) radial nerve is trapped in the fracture site d) ulnar nerve is compressed by hematoma e) median nerve is trapped in fracture site 2. A 55 year old female complains of numbness and discomfort in her right hand, which is awakening her from sleep. Similar symptoms occur while steering the car or writing. You suspect carpal tunnel syndrome. Which of the following would lead you to consider alternative diagnoses? a) absence of sensory loss on testing with pinprick b) absence of thenar muscle wasting c) absence of weakness of abductor pollicis brevis d) presence of rheumatoid arthritic changes in the hands e) wasting of the thenar, hypothenar and interosseous muscles of the right hand 3. Which antibiotic is the first line drug of choice for an otherwise uncomplicated human bite infection of the hand? a) Gentamicin b) Penicillin c) Flagyl d) Cephalexin e) Erythromycin 4. Which of the following is correct regarding hemangiomas? a) they usually will require surgery b) most present in the teenage years c) they usually resolve on their own d) most are found on the extremities e) they are composed mostly of dilated veins 5. All the following are common conditions seen with a rheumatoid hand EXCEPT: a) synovitis b) radial drift c) thumb deformity d) Swan neck deformity e) Boutonniere deformity 6. A 23 year old man is involved in a house fire, sustaining second and third degree burns to his face, anterior chest and trunk, entire left arm, groin, and both legs circumferentially. What is the estimated TBSA involved in the burn? a) 68.5% b) 67.5% c) 46% d) 55% 7. All of the following burns require extra fluid administration EXCEPT: a) burns greater than 70% TBSA b) electrical burn c) inhalation injury d) pediatric burns e) 4 burns 8. All of the following are complications of breast implants EXCEPT: a) capsular contraction b) increased risk of infection c) breast asymmetry d) systemic inflammatory reaction 9. All of the following statements are true concerning a hypertrophic scar EXCEPT: a) it frequently improves with corrective surgery and/or pressure b) it is more common in blacks c) it does not outgrow its border d) it occurs soon after surgery and usually subsides with time 10. Regarding fractures of the mandible, all of the following are correct, EXCEPT: a) they frequently are compound b) they produce numbness of the lip c) deformity and malocclusion are the usual presenting features d) the muscles of mastication keep fragments from displacement e) the majority can be treated by dental fixation 11. All of the following are true statements regarding wound contractures EXCEPT: a) contracture is due to dehydration of coagulum and is not an active, energy-dependent process b) contracture begins at about the fourth or fifth post wound day c) myofibroblasts move surrounding dermis inward d) contraction may be inhibited by cytotoxic drugs e) contracture occurs in an inflammatory milieu 12. During wound healing, collagen is laid down by: a) the endothelial cell b) the fibroblast c) the wandering macrophage d) the surrounding epithelium e) none of the above

MCCQE 2002 Review Notes

Sample Questions 39

PSYCHIATRY
1. Monoamine oxidase inhibitor drugs are used in the treatment of depression because they increase synaptic levels of: a) gamma-aminobutyric acid (GABA) b) histamine c) acetylcholine d) norepinephrine e) somatostatin 2. Neuropsychological effects of hallucinogens may include all of the following EXCEPT: a) miosis b) tremor c) hyper-reflexia d) incoordination e) blurred vision 3. Cocaine withdrawal can include all of the following EXCEPT: a) Crash sleep b) anergia c) anhedonia d) euphoria e) continued craving 4. Alcohol withdrawal includes all of the following EXCEPT: a) autonomic hyperactivity b) tremor c) starts within 2-4 hours after prolonged drinking d) nausea e) irritability 5. Which would not be considered a risk factor for suicide in patients presenting with suicidal ideation: a) substance abuse b) male gender c) lack of social supports d) unsuccessful attempt at suicide in the past e) childless marriage 6. A 54 year-old man has become forgetful, preoccupied, withdrawn, irritable and disheveled. His physical examination was normal. The patient had been with his company for twenty-two years and was considered an excellent employee. Which of the following is the most likely diagnosis: a) multi-infarct dementia b) hypothyroidism c) schizophrenia d) alcoholism e) major depression 7. Which of the following is correct about depression in children: a) family therapy should be avoided because it scape goats a child who is already vulnerable b) symptoms may manifest as antisocial behaviour c) antidepressants generally are not effective in children d) the suicide rate in children aged 8-13 is higher than it is in older adolescents e) depression in children has been shown to be a prodrome to the later development of schizophrenia 8. All of the following are classified as paraphilias EXCEPT: a) fetishism b) homosexuality c) exhibitionism d) sexual sadism e) transvestism 9. A 32 year old engineer has been uncharacteristically active for several weeks. He spends most of his time at work and gets little sleep. He has told another engineer that he is involved in a research project that will earn me the Nobel Prize. Expensive research equipment keeps arriving at his office. The engineer is irritable, and it is hard to hold his attention. A classmate from graduate school recalls that the patient behaved in a similar twice during stressful periods at school. Long-term drug therapy for this patient would likely include: a) haloperidol b) valproic acid c) clozapine d) ascorbic acid e) chlordiazepoxide 10. Elderly male depressives typically present with all of the following EXCEPT: a) importuning b) anxiety c) weight loss d) little suicide risk e) insomnia 11. From among the drugs listed below, which would be the cause for most concern in an overdose: a) paroxetine (SSRI) b) amitriptyline (tricyclic) c) diazepam (benzodiazepine) d) chlorpromazine (phenothiazine) e) fluoxetine (SSRI) 12. Which of the follow statements about schizophrenia is false? a) male schizophrenics to experience their first psychotic episode at a younger age than women b) male schizophrenics are more frequently hospitalized than female schizophrenics c) compared to young female schizophrenics, young male schizophrenics are at increased risk of movement disorders secondary to neuroleptics d) in women, the symptoms of schizophrenia tend to worsen after menopause e) all of the above statements are true 13. A 29 year old school teacher who lives alone is brought to the emergency room because she has become increasingly suspicious, hyperactive, and anorexic over the past two days. She believes that people in the neighbourhood are out to get me. She has not slept in 2 nights. She reports seeing snakes crawling on the wall. Based on this information, the most likely diagnosis of the womans problem is: a) anorexia nervosa b) cocaine withdrawal c) paranoid personality d) psychostimulant abuse e) shared paranoid disorder

40 Sample Questions

MCCQE 2002 Review Notes

PSYCHIATRY

. . . CONT. 20. Which of the following statements concerning anorexia nervosa and bulimia nervosa is false: a) patients with either of these eating disorders are preoccupied with weight, food, and body shape b) bulimia nervosa is more prevalent than is anorexia nervosa c) both of these eating disorders are more common in females than in males d) bulimia nervosa often presents earlier in adolescence than does anorexia nervosa e) bulimic symptoms may occur in both bulimia nervosa and anorexia nervosa 21. A pattern of unstable but intense interpersonal relationships, impulsivity, inappropriately intense anger, identity disturbance, affective instability, and problems with being alone suggest a diagnosis of: a) antisocial personality disorder b) narcissistic personality disorder c) histrionic personality disorder d) schizoid personality disorder e) borderline personality disorder 22. The criteria for diagnosis of a factitious disorder include: a) intentional production or feigning of physical signs or symptoms b) absence of secondary gain c) possibility of economic gain d) (a) and (b) e) (a) and (c) 23. A patient with a fear of heights is brought to the top of a tall building and required to remain there as long as necessary for the anxiety to dissipate. This is an example of: a) graded exposure b) participant modeling c) positive reinforcement d) flooding e) relationship therapy 24. Hypnosis has been used successfully in all of the following conditions EXCEPT: a) pain b) phobia c) paranoia d) anxiety e) smoking 25. Cognitive therapy helps to correct which of the following cognitive distortions: a) depersonalization b) psychotic thinking c) over-generalizations d) selective inference e) hallucinations 26. The most frequently reported side effect of tricyclic antidepressants is: a) peripheral neuropathy b) photosensitivity c) agranulocytosis d) jaundice e) dry mouth

14. Anti--1-adrenergic blockade causes: a) nausea b) constipation c) orthostatic hypotension d) dry mouth e) drowsiness 15. The following are common side effects of SSRIs EXCEPT: a) headache b) sexual dysfunction c) vomiting d) anorexia e) orthostatic hypotension 16. Clozapine is the neuroleptic of choice for schizophrenia when: a) the patient shows no evidence of tardive dyskinesia b) the patient has not improved with adequate dosages for 6 weeks of 3 other antipsychotics c) the patient is under fifteen years of age d) the patient has a WBC count of >10 000 e) the patient is very sensitive to anticholinergic side effects 17. Frequent conditions appearing comorbidly with ADHD include: 1) oppositional defiant and conduct disorders 2) anxiety disorder 3) learning disabilities 4) tic disorders 5) language disorders Which are correct: a) only 1 is correct b) 1, 2, and 3 are correct c) 2 and 4 are correct d) all are true 18. A 35 year old man presents to the emergency room with suicidal ideation. He describes significant stress due to recent job loss and financial hardship. Further inquiry reveals a history of repeated job loss, fraud charges, and frequent arm slashing to decrease stress. The man was not disruptive as a child. The most likely diagnosis is: a) borderline personality disorder b) antisocial personality disorder c) adjustment disorder d) dysthymic disorder e) schizophrenia 19. Which of the following statements concerning childhood abuse and neglect is false: a) the abuser is commonly a friend or a relative b) physical indicators of sexual abuse include bruising or pain in the genital region c) children who have been abused often present with aggressive or anxious behaviors d) Mmentally retarded or physically handicapped children may be at increased risk of abuse e) local child welfare authorities must be notified in cases of suspected child abuse, however there is no duty to report child neglect

MCCQE 2002 Review Notes

Sample Questions 41

PSYCHIATRY

. . . CONT. 33. Which of the following symptoms of schizophrenia responds best to anti-psychotic medication: a) asocial behavior b) flat affect c) paranoid delusions d) lack of motivation e) all of the above 34. A 53 year-old housewife presents with depression marked by early morning wakening, diminished energy and poor concentration. She is treated with supportive psychotherapy and imipramine 100 mg qhs. After four weeks she is no better. You would next: a) reassure her that antidepressants take 5-6 weeks to be effective b) begin intensive psychotherapy c) switch to a serotonin reuptake inhibitor d) increase the dose to 150 mg qhs e) add lithium carbonate 1200 mg daily 35. Hypertensive encephalopathy is a serious complication of treatment with: a) phenothiazines b) tricyclic antidepressants c) lithium carbonate d) MAOI antidepressants e) benzodiazepines 36. Electroconvulsive therapy (ECT) is a first line treatment in which one of the following conditions? a) obsessive compulsive disorder b) paranoid schizophrenia c) generalized anxiety disorder d) acute mania e) major depression with acute suicidal ideation 37. Tardive dyskinesia is: a) an acute extrapyramidal side effect of phenothiazines b) an acute anticholinergic side effect of tricyclic antidepressants c) a type of Parkinsons disease d) a side effect of excessive ECT treatments e) a long term complication of chronic phenothiazine administration 38. Which of the following symptoms is most commonly found in schizophrenia? a) depressed mood b) flight of ideas c) elevated mood d) delusional fears e) thought insertion 39. Patients with conversion disorders will show each of the following EXCEPT: a) La belle indifference b) severe depression c) loss of special sense function d) secondary gains e) paralysis of voluntary muscles

27. A high risk of suicide is associated with which one of the following factors: a) female aged less than 30 years b) married male less than 30 years c) public setting d) secondary gain from attempt e) single male aged more than 60 years 28. A fixed unalterable belief that is false in its content and in its social and cultural setting is called: a) an illusion b) a hallucination c) a delusion d) agnosia e) paranoia 29. Criteria for involuntary hospitalization of a suicidal patient include: a) assessment done less than 72 hours prior to completion of Form 1 b) presence of a mental disorder c) acute risk of harm to themselves d) (a), (b), and (c) e) (b) and (c) 30. Each of the following statements about affective disorders is true EXCEPT: a) patients with bipolar disorder show roughly the same frequency of positive family history as do patients with unipolar disorder b) major depressive illness is more common in women than in men c) bipolar affective disorder is far less common than unipolar affective disorder d) there is a different response to lithium in unipolar and bipolar disorder e) imipramine is more likely to produce hypomania in bipolar patients than in unipolar patients 31. A major depressive disorder is diagnosed only when the affective disturbance has existed for at least : a) two weeks b) four weeks c) six weeks d) eight weeks e) twelve weeks 32. A 30 year-old man presents in emergency with right lower quadrant abdominal pain. His wife reports that he had been drinking heavily in response to marital problems and had never had such pain before. Appendicitis was diagnosed and an appendectomy was successfully performed. Four days later the patient was anxious, restless, unable to sleep and claimed his wife was a stranger trying to harass him. The likeliest diagnosis is: a) paranoid reaction b) delirium tremens c) mania d) schizophreniform reaction e) post-operative delirium

42 Sample Questions

MCCQE 2002 Review Notes

PSYCHIATRY

. . . CONT. 48. Which of the following investigations need not be performed before starting treatment with lithium carbonate? a) serum creatinine b) serum electrolytes c) thyroid function studies d) serum bilirubin e) all of the above 49. All of the following are common side effects of tricyclic antidepressants EXCEPT: a) dry mouth b) constipation c) parkinsonian tremor d) tachycardia 50. The Theory of Behavioral Therapy employs which of the following concepts: a) reaction formation b) reinforcement c) imprinting d) autosuggestion e) sublimation 51. Which of the following represents a contraindication for psychoanalysis? a) the existence of transference feeling toward the analyst b) the existence of countertransference feeling in the analyst c) the resistance to change by the patient d) preoccupation with resolving a crisis situation e) the existence of an underlying neurotic style of behaviour 52. Which of the following is least helpful in treating depression? a) phenelzine b) imipramine c) lithium carbonate d) lorazepam e) all of the above 53. A young mother is very focused on the health of her 16 month-old. She keeps her house immaculate for fear that dirt will harm her baby, she checks the lock on the door at least ten times before retiring to bed, and she has to get up and check that her child is still breathing at least 3 times every night. She knows that her fears are irrational but persists with these behaviors. The most likely diagnosis is: a) paranoid delusions not otherwise specified b) post-partum depression c) obsessive compulsive disorder d) generalized anxiety disorder e) paranoid personality disorder 54. A thorough assessment for the presence/absence of alcohol withdrawal should include questions about all of the following EXCEPT: a) nausea and vomiting b) mood c) difficulty walking (ataxic gait) d) visual disturbances e) tremulousness

40. The syndrome of delirium tremens is associated with each of the following EXCEPT: a) rapidly fluctuating level of consciousness b) dehydration c) visual hallucinations d) renal failure e) polyneuropathy 41. Which of the following is best treated with high dose benzodiazepines: a) schizophrenia, catatonic type b) major depression c) generalized anxiety disorder d) delirium tremens e) psychogenic amnesia 42. Personality types predisposed to depression include which one of the following: a) dependent b) antisocial c) schizoid d) paranoid e) schizotypal 43. Which of the following neurological symptoms can be produced by antipsychotic drugs? a) akathesia b) shuffling gait c) oculogyric crisis d) tremor at rest e) all of the above 44. Absolute contraindications to ECT include: a) pregnancy b) recent myocardial infarction c) fractured pelvis d) brain tumor e) all of the above 45. In the elderly delirium may be produced by the use of: a) neuroleptics b) tricyclic antidepressants c) antiparkinsonian agents d) minor tranquilizers e) all of the above 46. Which of the following statements about simple phobia is correct? a) it responds well to simple reassurance b) it responds well to individual psychotherapy c) it responds well to benzodiazepines d) it responds well to relaxation and desensitization e) all of the above 47. Toxicity due to lithium carbonate is associated with all of the following EXCEPT: a) nausea b) serum lithium level of 0.1 mEq/L c) tremulousness d) convulsions

MCCQE 2002 Review Notes

Sample Questions 43

PSYCHIATRY

. . . CONT. 63. Which of the following statements is true about paraphilias: a) almost never diagnosed in women b) treated with androgen drugs c) includes: exhibitionism, fetishm, voyeurism, pedophilia d) a and b e) a and d 64. All of the following statements are correct EXCEPT: a) female orgasmic disorder can be caused by denervation of the lumbosacral spine a) having a rigid, conservative sexual upbringing can result in delayed ejaculation in men a) phenothiazines are used to treat delayed ejaculation disorder in males a) most delayed ejaculation is situational a) most orgasmic disorders in women are psychological. 65. Major Depressive Disorder with melancholic features is most likely accompanied by: a) echololia and /or echopraxia b) hypersomnia c) severe anhedonia d) weight gain e) inappropriate posturing 66. Long acting antipsychotic medications: a) are available in both typical and atypical preparations b) are administered IM or SC c) are first line treatment for severe schizophrenia d) are less likely to cause EPS compared to the oral form e) should be ideally started after the patient has had exposure to the oral form 67. A 55-year-old man with chronic schizophrenia is brought to the E.R. He is diaphoretic, pale and has a temperature of 41C, blood pressure of 170/100, and heart rate of 120. He also exhibits muscle rigidity and a fluctuating level of consciousness. The most diagnosis is: a) acute dystonic reaction b) tardive dyskinesia c) neuroleptic malignant syndrome d) psychosis secondary to a general medical condition e) substance-induced psychosis 68. Obsessive-compulsive disorder is most consistent with: a) perfectionism, meticulous organization and cleanliness a) the persistent need to shower because of the belief that aliens are contaminating ones body b) recurrent and intrusive thoughts that are inserted into ones mind by others c) a persistent preoccupation with food and ones appearance accompanied by bingeing, purging and/or restricting behaviours d) compulsively avoiding cracks in the side walk for fear that if one does not, a relative might be subject to harm 69. Bipolar disorder may be characterized by: a) rapid cycling: at least one episode (manic or depressed) every 4 weeks b) an increased risk in females c) a lower risk in higher socioeconomic groups due to decreased psychosocial stressors d) hypomanic episodes without depressed episodes, as in Bipolar II disorder e) none of the above

55. Which of the following will be least helpful with respect to distinguishing delirium from dementia: a) disorientation at night b) duration of disorientation c) fluctuating level of consciousness d) mini mental status examination e) presence of visual hallucinations 56. Onset of schizophrenia is typically in an affected individuals: a) early teens b) late teens/early 20s c) late 20s/early 30s d) late 30s/early 40s e) late 40s/early 50s 57. Which of the following is an example of a negative symptom of schizophrenia: a) nihilistic delusion b) flat affect c) auditory hallucinations d) catatonic behaviour e) all of the above 58. Helpful treatments for post-traumatic stress disorder symptoms include the following except: a) SSRIs b) relaxation techniques c) exposure therapy d) cognitive restructuring e) all of the above may be helpful 59. The primary difference between factitious disorder and malingering is: a) factitious disorder involves the feigning of symptoms for external rewards (e.g. economic gain) b) factitious disorder involves the unintentional production of symptoms c) there is an absence of primary gain in factitious disorder d) factitious disorder can involve only psychological symptoms e) there is an absence of secondary gain in factitious disorder 60. The following features tend to differentiate atypical antipsychotics from typical/conventional antipsychotics EXCEPT: a) atypicals are serotonin-dopamine antagonists b) atypicals tend to result in more specific dopamine blockade c) atypicals tend to result in lower levels of prolactin d) atypicals tend to cause fewer extrapyramidal side effects e) atypicals tend to be more effective for the treatment of negative symptoms 61. Which of the following statements is true about sexual dysfunction: a) it is secondary to physical factors alone b) it occurs only in females c) the commonest cause is medications d) psychological factors account for 10% in women e) none of the above 62. Which of the following statements is true about premature ejaculation: a) most common male sexual dysfunction b) usually secondary to performance anxiety c) has a 90% success rate with regards to treatment d) treatment includes: excercises to focus on experience vs performance, increasing stimulation and control exercises e) all of the above

44 Sample Questions

MCCQE 2002 Review Notes

RESPIROLOGY
1. which of the following is an incorrect statement regarding transudative pleural effusions? a) pleural LDH/serum LDH is > 0.6 b) pleural protein/serum protein is < 0.5 c) pelural LDH <2/3 upper limit of normal serum LDH d) all of the above e) none of the above 2. Which of the following organisms causes walking pneumonia? a) S. aureus b) S. pneumoniae c) H. fluenzae d) Mycoplasma e) Legionella 3. Which of the following is considered a positive tb skin test? a) induration of > 5 mm in close contact with an HIV positive individual b) induration of < 5 mm in an adult with no risk factors c) induration of < 5 mm in a person with silicosis d) induration of > 5 mm in a person with known TB exposure 4. The following are typical chest x-ray findings of a malignant solitary nodule: a) > 3 cm b) ill-defined margins c) popcorn pattern of calcification d) associated pleural effusions e) cavitation with thick wall 5. All of the following but one are indicative of poorly controlled asthma: a) night time awakenings b) use of a beta-agonist 2x/week c) mild limitation of activities d) chronic dry cough 6. All of the following but one are typical chest x-ray findings in a person with COPD: a) hyperinflation b) flat hemidiaphragm c) increased AP diameter d) increased retrosternal air space e) increased peripheral markings 7. Which of the following is NOT an indication of home O2 in a person with COPD? a) PaO2 < 55 mmHg b) PaO2 < 60 mmHg with erythrocytosis or cor pulmonale c) persistant hypoxemia after 3 weeks of maximal therapy d) PaO2 maintained > 80 mmHg during wakeful res increased by 250 ml during exercise or sleep 8. Which of the following are associated with upper lobe fibrosis? a) farmers lung b) silicosis c) sarcoid d) histiocytosis e) X asbestos 9. The differential diagnosis of obstructive lung disease includes: a) COPD b) asthma c) bronchiectasis d) CF e) chest wall disease 10. Which is NOT a feature of adenocarcinoma of the lung? a) solitary peripheral pulmonary nodule b) may appear as an alveolar infiltrate simulating pneumonia c) frequently unilateral d) metastasizes early

MCCQE 2002 Review Notes

Sample Questions 45

RHEUMATOLOGY
1. A 21 year-old bisexual man has a 4 week history of intermittent diarrhea, urethral discharge, and pain in the right knee and left second toe. He has several oral ulcers, a clear urethral discharge, a scaly papular rash on palms and soles, onycholysis, sausage-like swelling of the left second toe, and heat and swelling of the right knee. The results of Gram stains and cultures of urethral discharge are negative. Rheumatoid factor is not present. The most likely diagnosis is: a) Reiters syndrome b) gonococcal arthritis c) Behets disease d) acquired immune deficiency syndrome e) psoriatic arthritis 2. A 53 year-old presents to your office with pain and stiffness in both hands and knees of 6 months duration. Which of the following findings on your physical examination may help with a diagnosis: a) joint tenderness/effusions b) maculopapular rash c) iridocyclitis d) hepatosplenomegaly e) all of the above 3. Radiographic features of osteoarthritis of the knee include which of the following: a) marginal erosions b) juxta-articular osteopenia (demineralization) c) loss of articular cartilage with narrowing of the radiologic joint space d) osteonecrosis (avascular necrosis) of the medial femoral condyle e) high riding patella (patella alta) 4. Which of the following is true about serologic testing in SLE? a) A positive ANA is specific for SLE b) ds-DNA level correlates with disease activity in SLE c) Anti-histone antibodies are seldom positive in non-drug induced SLE d) The majority of patients with SLE have anti-Sm antibodies e) Anti-Ro antibody is specific for SLE 5. The treatment of choice for thrombotic events in the antiphospholipid antibody syndrome is: a) intravenous steroids b) high-dose oral steroids with a rapid taper c) penicillamine d) aspirin e) warfarin 6. A 70 year-old woman presents with acute knee arthritis. Radiographs show meniscal calcification (chondrocalcinosis). Analysis of the synovial fluid reveals weakly positive birefringent rhomboid-shaped crystals. The crystals are most likely: a) monosodium urate b) calcium hydroxyapatite c) cholesterol d) calcium pyrophosphate dihydrate e) dicalcium phosphate dihydrate (Brushite) 7. Which of the following is not a disease modifying agent for rheumatoid arthritis? a) infliximab b) gold c) high dose corticosteroid d) methotrexate e) hydroxychloroquine 8. A 25-year-old male presents to the emergency room with a 6-week history of progressive weakness. On questioning, the man has difficulty combing his hair, brushing his teeth, and climbing stairs. Night sweats, fever and malaise are present. He has noticed a purplish swelling around his eyes. Bloodwork shows a high AST and ALT. ALP is normal. High on your differential diagnosis is: a) rheumatoid arthritis b) trauma-related back pain c) inflammatory myopathy d) polyarteritis nodosa e) polymyalgia rheumatica 9. A 39-year-old female presents to your office with an insidious onset of right knee pain. The differential diagnosis includes: a) synovitis b) degenerative arthritis c) ligament derangement d) bursitis e) all of the above 10. Systemic lupus erythematosus may be characterized by all of the following EXCEPT: a) hemolytic anemia b) discoid rash c) serositis d) erosive arthritis e) psychosis 11. Physical examination manoeuvres specific to inflammatory spondyloarthropathies include all of the following EXCEPT: a) straight leg raising b) occiput-wall distance c) chest expansion d) Schebers test

46 Sample Questions

MCCQE 2002 Review Notes

RHEUMATOLOGY

. . . CONT. 19. First line pharmacologic treatment for osteoarthritis is: a) acetaminophen 1000 mg po QID b) celecoxib 200 mg po BID c) glucosamine 500 mg po QID d) ibuprofen 400 mg po TID e) depo-medrol 20 mg intra-articularly 20. Which of the following medications is not associated with drug-induced lupus syndrome? a) isoniazid b) propranolol c) hydralazine d) cyclosporine d) carbamazepine 21. A 30-year-old woman comes to your office with a 1-day history of right knee pain and swelling. She has never had joint pain. She spent the day before working in her garden, and was kneeling for most of the time. She has no other associated symptoms, but has had several sexual partners within the last year. You aspirate her knee because you are worried about septic arthritis and send requisitions for cell counts, microscopy, stat Gram stain, glucose, LDH and culture. The lab phones you back in the next hour with the following results: WBC 40,000 % PMN 35% Stat Gram stain negative Crystals negative Glucose 1.1mmol/L What is your next step? a) Send the patient home and advise her to take extrastrength acetaminophen - this is probably traumatic b) Advise her to go to the emergency room with a letter instructing the emergency staff to draw blood cultures and start antibiotics c) Give the patient intra-articular steroids d) Send the patient home and ask her to return to your office tomorrow e) Refer her to a rheumatologist

12. Cautions and contraindications of NSAIDs include: a) a creatinine of 180 mmol/L b) peptic ulcer disease c) warfarin use d) cirrhosis e) all of the above 13. Small vessel vasculitides include all of the following EXCEPT: a) Wegeners granulomatosis b) Churg-Strauss syndrome c) essential cryoglobulinemia vasculitis d) Kawasakis disease e) predominantly cutaneous vasculitis 14. A 30-year-old male presents with a history of mild upper respiratory tract infection progressing to a severe cough and hemoptysis. Upon examination, you note a nasoseptal perforation. Tests you should order include: a) chest x-ray b) urine routine and microscopy c) CBC d) c-ANCA e) all of the above 15. A 50-year-old female presents with headache, scalp tenderness and painless loss of vision in her right eye. Your initial management is: a) temporal artery biopsy b) prednisone 1mg/kg c) infliximab infusion d) cyclophsophamide 2mg/kg e) consult to Ophthalmology 16. Psoriatic arthritis may be characterized by: a) subungual hyperkeratosis b) DIP involvement c) conjunctivitis d) dactylitis e) all of the above 17. All of the following statements about septic arthritis are true EXCEPT: a) permanent joint damage can quickly occur b) N. gonorrhea accounts for 75% of cases in young adults c) Salmonella species characteristically infect those with idiopathic thrombocytopenic purpura d) predisposing factors include prior antibiotic use e) IV antibiotics should be started empirically 18. Cauda equina syndrome is characterized by the following EXCEPT: a) saddle anesthesia b) urinary retention c) weak plantar flexion d) decreased anal sphincter tone e) posterior thigh pain

MCCQE 2002 Review Notes

Sample Questions 47

UROLOGY
1. An 84 year old man presents to your office complaining of a five day history of passing terminal bright red bloody urine. He reports marked urgency and frequency but is otherwise well and afebrile. In retrospect, he remembers being quite ill with an infection in the recent past. Analysis of a voided specimen shows 3 smooth and rounded RBCs/hpf. Which of the following is NOT included in your differential diagnosis? a) benign prostatic hpertrophy (BPH) b) post-streptococcal glomerulonephritis c) a stone lodged at the vesico-ureteric junction d) a superficial bladder tumour e) a coliform positive UTI 2. Of the following conditions, which is not considered a urological emergency? a) paraphimosis b) priapism c) orchitis d) testicular torsion e) autonomic dysreflexia 3. A normal semen analysi includes all of the following EXCEPT: a) pH between 7.5-8.1 b) a sperm count > 20 million sperm/ml c) WBC < 10/hpf d) motility > 50% e) > 60% normal morphological forms 4. A mother brings her toilet-trained, 5 year old daughter to see you with a 3 day history of dysuria, increased frequency, and urgency incontinence. Her past medical history is significant for a bag-specimen positive UTI charaterized by high fever and vomiting at 2 years of age. At the time, she was lost to follow up. Which of the followin would be appropriate for your initial management of this patient? a) voiding cystourethrogram (VCUG) b) a mid stream urine specimen c) nuclear cystogram d) DMSA renal scan e) abdominal ultrasound 5. A 55 year old man reluctantly admits to difficulty in maintaining an erection sufficient for sexual intercourse for the past two years. He reports being healthy, however, he has not seen a physician in 10 years. Which of the following would not suggest a potential cause for his erectile dysfunction? a) elevated blood pressure b) galactorhea c) acanthosis nigracans d) varicocele e) eruptive xanthoma 6. A 50 year old man presents with a 2 month history of irritative voiding symptoms and a fever. He initially saw his family doctor who treated him for cystitis with ciprofloxacin for 5 days. He now presents to you because his symptoms have returned soon after he finished his course of antibiotics. All of the following are likely TRUE except: a) he never had a UTI b) he might have complained of discomfort when riding a bicycle c) he might have also experienced back pain d) this patient does not need to be investigated e) he could of had prostatitis and should have been treated at least 3 weeks

48 Sample Questions

MCCQE 2002 Review Notes

ANSWERS TO SAMPLE QUESTIONS


ANESTHESIA
1. B 11. A 2. B 12.D 3. D 13. C 4. C 14. C 5. B 6. A 7. B 8. E 9. C 10. C

CARDIAC AND VASCULAR SURGERY


1. 2. 3. 4. 5.

CARDIOLOGY
1. B 2. B 3. C 4. D 5. C 6. B 7. D 8. E 9. D 10. A

COMMUNITY HEALTH
1. B 11. B 2. D 12.E 3. D 13.E 4. A 14. A 5. D 15. C 6. D 16. D 7. E 17. C 8. A 18.E 9. B 10. A

DERMATOLOGY
1. 11. 21. D 2. 12. 3. 13. 4. 14. 5. 15. 6. 16. 7. 17. 8. 18. 9. 19. 10. 20.

EMERGENCY MEDICINE
1. C 11. B 2. C 12.B 3. B 13.B 4. D 14. E 5. D 15. D 6. E 7. E 8. B 9. C 10. B

ENDOCRINOLOGY
1. B 11. D 21. E 2. D 12.C 22.A 3. C 13.C 4. A 14. A 5. E 15. B 6. C 16. B 7. B 17. C 8. B 18.B 9. C 19. E 10. C 20. B

FAMILY MEDICINE
1. C 11. D 2. D 12.E 3. B 13. B 4. E 14. C 5. A 6. C 7. D 8. A 9. C 10. B

GASTROENTEROLOGY
1. B 11. A 2. A 12.B 3. D 13.C 4. C 14. A 5. B 15. A 6. C 16. B 7. E 8. B 9. C 10. D

MCCQE 2002 Review Notes

Sample Questions 49

ANSWERS TO SAMPLE QUESTIONS . . . CONT.


GERIATRIC MEDICINE
1. B 2. D 3. E 4. B 5. A

GYNECOLOGY
1. B 2. C 3. B, C, E, F, G, H, I 9. A, E, F 15.E 4. B, C, D, E, F, G, H 10. A, B, C, D, F, G, H, I 5. 1, B, C, D, E, G 11. E 6. B, C, F 12. A 7. A, B, E, G 8. B, C, G 13. D 14. E

HEMATOLOGY
1. B 2. D 3. E 4. B 5. A

INFECTIOUS DISEASES
1. D 11. B 2. A 12.C 3. C 13.B 4. D 14. A 5. B 15A 6. E 7. B 8. D 9. C 10. E

NEPHROLOGY
1. A 11. D 2. C 12.B 3. C 13.B 4. D 14. A 5. C 15. D 6. B 7. A 8. A 9. B 10. B

NEUROLOGY
1. B 11. E 2. D 12.D 3. A 4. C 5. C 6. D 7. A 8. E 9. C 10. C

NEUROSURGERY
1. B 11. D 2. D 12.D 3. B 13.C 4. A 14. B 5. C 15. A 6. D 7. B 8. A 9. B 10. C

OBSTETRICS
1. D 11. A 2. D 12.D 3. E 13.A 4. E 14. D 5. B 15. D 6. E 7. A 8. A 9. A 10. A

OPHTHALMOLOGY
1. B 11. A 21.A 2. A 12.C 22.C 3. B 13.D 23.B 4. C 14.B 24.E 5. D 15.A 6. B 16.C 7. D 17.D 8. A 18.C 9. C 19.B 10. C 20.C

ORTHOPEDICS
1. B 2. C 3. D 4. B 5. E

50 Sample Questions

MCCQE 2002 Review Notes

ANSWERS TO SAMPLE QUESTIONS . . . CONT.


OTOLARYNGOLOGY
1. C 11. B 21. C 2. B 12.B 22.C 3. C 13.C 23.B 4. E 14. D 24. D 5. B 15. C 25. B 6. B 16. A 26. E 7. D 17. B 27. C 8. C 18.D 28.D 9. C 19. C 10. A 20. B

PEDIATRICS
1. E 11. D 21. E 31. D 41. A 51. A 2. A 12.B 22.B 32.B 42.B 52.D 3. A 13.D 23.D 33.A 43.B 53.B 4. B 14. B 24. C 34. C 44. E 54.B 5. C 15. B 25. B 35. C 45. D 55. A 6. B 16. B 26. C 36. A 46. A 56. E 7. E 17. B 27. C 37. C 47. B 57. E 8. A 18.C 28.B 38.E 48.D 58.D 9. B 19. A 29. C 39. D 49. A 10. E 20. A 30. C 40. C 50. D

PHARMACOLOGY
1. C 11. 2. C 12.D 3. C 4. D 5. C 6. E 7. B 8. D 9. E 10. D

PLASTIC SURGERY
1. C 11. A 2. E 12.B 3. B 4. C 5. B 6. A 7. A 8. D 9. B 10. D

PSYCHIATRY
1. D 11. B 21. E 31. A 41. D 51. D 61. C 2. A 12.E 22.E 32.B 42.A 52.D 62.E 3. D 13.D 23.D 33. C 43.E 53.C 63.A 4. C 14. C 24. C 34. D 44. D 54. C 64. C 5. E 15. E 25. E 35. D 45. E 55. A 65. C 6. E 16. B 26. E 36. E 46. D 56. B 66. E 7. B 17. D 27. E 37. E 47. B 57. B 67. C 8. B 18.A 28.C 38.D 48.D 58.E 68.E 9. C 19. E 29. E 39. B 49. C 59. C 69. E 10. D 20. D 30. A 40. D 50. B 60. B

RESPIROLOGY
1. A 2. D 3. A 4. C 5. B 6. E 7. D 8. E 9. E 10. C

RHEUMATOLOGY
1. A 11. A 21. B 2. E 12.E 3. C 13.D 4. B 14. A 5. E 15. B 6. D 16. E 7. C 17. C 8. C 18.E 9. E 19. A 10. D 20. D

UROLOGY
1. B 2. C 3. A 4. A 5. D 6. D

MCCQE 2002 Review Notes

Sample Questions 51

KEY FEATURE QUESTIONS


Instructions for Examinees
Key Feature questions can pertain to solutions of clinical cases that involve data-gathering (e.g., history taking, physical examination, laboratory investigations), diagnosis or treatment. The Key Feature questions will ask for your answers in different ways: you may be asked to (i) select the correct response(s) from a menu of options, or (ii) print the correct answer(s) on a blank line. Please note the following guidelines before proceeding: 1. After reading the description of the case, read the question carefully. Take note as to whether you are told the maximum number of answers to provide. If you exceed this number, you will receive no credit for the question. If you select or list fewer answers, you will still receive credit for any of your answers which are correct. If you are asked to select your answers from a menu: a. Read the complete menu of options before recording your answers. b. Make certain that the number of answers you record is not greater than the number you have been asked to select. If you are asked to list your answers below the question: a. Ensure that you print each answer legibly. b. Record each answer on a separate blank line. c. Do not record more than the maximum number you are asked to list.

2.

3.

CASE1
A 30 year-old homeless male presents with a 4 week history of feeling generally unwell with anorexia, weight loss and intermittent fever. He lives primarily on the streets or occasionally in a mens shelter. He admits to regular use of intravenous heroin for the last seven years and often shares needles.

QUESTION 1 (CASE 1)
Given this history, which of the following diagnoses would you consider? Select up to five. 1. Acute pancreatitis 2. AIDS related complex 3. Aspergillosis 4. Bacterial endocarditis 5. Chronic pancreatitis 6. Delirium tremens 7. Diabetes mellitus 8. Hepatitis B 9. Hepatocellular carcinoma 10. Histoplasmosis 11. Leukemia 12. Lymphoma 13. Osteomyelitis 14. Peptic ulcer disease 15. Pericarditis 16. Pyelonephritis 17. Renal failure 18. Secondary syphilis 19. Talc pneumonitis 20. Tuberculosis

QUESTION 2 (CASE 1)
A mitral regurgitant murmur is auscultated in this patient. His temperature is 39.0C. Which of the following other findings may be found on physical examination given the diagnoses being considered? Select up to five. 1. Ascites 2. Campbell De Morgan spots 3. Cannon a waves 4. Cyanosis 5. Erythema nodosum 6. Follicular keratosis 7. Gynecomastia 8. Hepatomegaly 9. Janeway lesions 10. Jaundice 11. Koilonychia 12. Livedo reticularis 13. Osler nodes 14. Pallor 15. Palmar erythema 16. Papilledema 17. Purpura 18. Roth spots 19. Spider nevi 20. Splenomegaly

52 Sample Questions

MCCQE 2002 Review Notes

KEY FEATURE QUESTIONS


CASE 2

. . . CONT.

CASE 3
An 84 year-old female is brought to the Emergency Department by her daughter who states that her mother has not been herself over the past three days. Further questioning reveals that the mother has been less attentive than usual and more withdrawn, her conversation is often rambling and she has not been sleeping well. Prior to this, she functioned very well.

A babysitter brings a 14-month-old to emergency because of inconsolable crying. The child appears well cared for and healthy but a "bruise" is noted on the left humerus and a well-circumscribed small fresh burn is noted on the other shoulder.

QUESTION 3 (CASE 2)
List four well recognized risk factors for child abuse: 1. _____________________________________________________________ 2. _____________________________________________________________

QUESTION 6 (CASE 3)
What diagnoses would you consider at this time? List up to two. 1. _____________________________________________________________

3. _____________________________________________________________ 2. _____________________________________________________________ 4. _____________________________________________________________

QUESTION 4 (CASE 2)
Which of these physical findings are most suggestive of child abuse? Select up to four. 1. Alopecia 2. Blue sclera 3. Bruises on shins and elbows 4. Buttocks wasting 5. Caput medusa 6. Clubbing 7. Condylomata acuminata 8. Eczema 9. Full fontanelle 10. Healed laceration on chin 11. Hutchison teeth 12. Impetigo 13. Large purple macular lesion on buttock 14. Limp 15. Oral thrush 16. Petechiae 17. Pitted finger nails 18. Proptosis 19. Retinal hemorrhage 20. Seborrhea

QUESTION 7 (CASE 3)
What additional aspects of history would be most important to know? Select up to four. 1. Alcohol use 2. Complete psychiatric history 3. Family history of Alzheimers disease 4. Financial status 5. History of fever 6. History of incontinence 7. History of osteoarthritis 8. History of Parkinsons disease 9. History of previous CVA 10. Immunization status 11. Marital status 12. Medication history 13. Occupational history 14. Pattern of symptoms over the course of the day, e.g. fluctuations 15. Presence of hallucinations 16. Previous history of depression 17. Previous seizure disorder 18. Previous surgery 19. Recent travel 20. Smoking history

QUESTION 5 (CASE 2)
You suspect this is a case of child abuse. Which of the following are appropriate steps in the initial management. Select up to four. 1. Abdominal ultrasound 2. Ascorbic acid level 3. Bone density studies 4. CBC 5. Chest x-ray 6. Coagulation studies 7. Creatinine 8. Dietary history 9. ECG 10. Factor VIII assay 11. Liver/spleen scan 12. MRI 13. Notify CAS (Childrens Aid Society) 14. Notify police 15. Notify public health nurse 16. Ophthalmology consult 17. Psychiatry consult 18. Serum calcium, phosphorus and alkaline phosphorus 19. Skeletal survey 20. Urinalysis

QUESTION 8 (CASE 3)
The interview confirms the daughter's description of the mother. Physical exam reveals: BP 120/80 supine and 100/70 sitting, RR=22, P=104, T=38.7C. JVP is at the sternal angle. Respiratory exam reveals dullness, increased tactile fremitus, crackles and bronchial breath sounds all in the left base. Heart sounds are normal. The abdominal exam is normal. CNS exam does not reveal any focal findings. What would you include in your initial investigations? Select up to four. 1. ALT 2. AST 3. Blood cultures 4. B12 5. CBC 6. Chest x-ray 7. CPK 8. CT scan of head 9. Drug screen 10. EEG 11. Electrolytes 12. HIV test 13. MRI of head 14. Protein electrophoresis 15. PT and PTT 16. RBC folate 17. SPECT scan 18. TSH 19. Urea and creatinine 20. VDRL

MCCQE 2002 Review Notes

Sample Questions 53

KEY FEATURE QUESTIONS


CASE 4

. . . CONT.

CASE 5
A 30 year-old woman comes to your office with a 3 day history of vulvar and vaginal pruritus.

A 57 year-old man presents to the Emergency Department with a 12 hour history of left flank pain.

QUESTION 12 (CASE 5) QUESTION 9 (CASE 4)


What diagnoses would you consider at this time? Select up to four. 1. Ruptured abdominal aortic aneurysm 2. Bowel obstruction 3. Appendicitis 4. Pyelonephritis 5. Diverticulitis 6. Renal calculi 7. Biliary colic 8. Renal cell carcinoma 9. Pneumonia What diagnosis would you consider at this time? List up to three. 1. _____________________________________________________________ 2. _____________________________________________________________ 3. _____________________________________________________________

QUESTION 13 (CASE 5)
What features on the history would you inquire about to make a diagnosis? Choose up to seven. 1. Date of last menstrual period 2. Sexual history 3. Vaginal discharge 4. Vulvar erythema 5. Vaginal odor 6. Recent antibiotic therapy 7. Diabetes 8. Macular rash 9. Psychiatric history 10. Headache 11. Allergies 12. Dysuria 13. Use of douches 14. Sore throat 15. Previous history of STDs

QUESTION 10 (CASE 4)
On history and physical exam, what symptoms and signs might aid in diagnosis? Choose up to seven. 1. Fever/chills 2. Weight loss 3. Rebound tenderness 4. Costovertebral angle tenderness 5. Urgency and frequency 6. Pruritus 7. Hypotension 8. Pulsatile abdominal mass 9 Nausea and vomiting 10. Left flank mass 11. Productive cough 12. Abdominal guarding 13. Pale stools and dark urine 14. Hematuria

QUESTION 14 (CASE 5)
What tests would you do to confirm the diagnosis? Choose up to four. 1. Urine C&S 2. Wet prep of vaginal discharge for microscopy 3. Urinalysis 4. CBC 5. ESR 6. Culture for gonorrhea 7. Culture for chlamydia 8. Abdominal CT 9. Pelvic ultrasound 10. Culture for Trichomonas vaginalis

QUESTION 11 (CASE 4)
What tests might be necessary to confirm your diagnosis? Choose up to six. 1. ESR 2. Serum amylase 3. Urine R&M 4. Chest x-ray 5. Abdominal/pelvic ultrasound 6. 3 views of the abdomen 7. Urine C&S 8. Abdominal CT 9. IVP 10. Total bilirubin 11. Electrolytes 12. 24 hour creatinine clearance 13. KUB 14. Serum Mg 15. Serum alkaline phosphatase 16. Barium enema 17. Serum Ca 18. CBC

CASE 6
A 65 year-old man in the emergency department has RUQ pain. He is otherwise healthy.

QUESTION 15 (CASE 6)
List three diagnoses you should consider in this patient. 1. _____________________________________________________________ 2. _____________________________________________________________ 3. _____________________________________________________________

54 Sample Questions

MCCQE 2002 Review Notes

KEY FEATURE QUESTIONS


QUESTION 16 (CASE 6)

. . . CONT.

QUESTION 20 (CASE 8)
With respect to the diagnosis, which elements of the history and physical will be most important in determining the etiology of this babys distress? Select up to four. 1. Delivery mode (C-section, vaginal) 2. Feeding pattern 3. Grunting 4. Head circumference 5. Heart sounds 6. History of maternal diabetes 7. Indrawing 8. Maternal temperature 9. Meconium stained amniotic fluid 10. Oxygen requirement 11. Presence of bruising 12. Presenting part (breech, vertex) 13. Previous obstetric history 14. Tachypnea 15. Vital signs

What are the most important questions to ask about on history? Choose up to seven. 1. Pruritus 2. Pale stools 3. Fever/chills 4. Hematuria 5. Jaundice 6. Anorexia 7. Dark urine 8. Weight loss 9. Back pain 10. Diarrhea 11. Alcohol/drug history 12. Nausea and vomiting 13. Fatigue 14. Cold intolerance

CASE 7
A 60 year-old woman comes to your office because she has seen bright red blood in the toilet bowl after bowel movements for the past month. She is otherwise healthy.

CASE 9
A 45 year-old migrant farm worker comes to see you with sudden onset of severe pain in his back and right leg after lifting some bales of hay. The pain radiates into the lateral aspect of the right foot and is associated with numbness along the lateral aspect of the foot. He has had a 5 year history of lower back pain with a similar episode of pain 1 year ago which was concentrated at the posterolateral aspect of the right calf and lateral aspect of the right foot. It became progressively worse, then resolved over a 3 week period.

QUESTION 17 (CASE 7)
What are three common diagnoses you should consider? 1. _____________________________________________________________ 2. _____________________________________________________________ 3. _____________________________________________________________

QUESTION 21 (CASE 9)
What elements of the history would you ask about in order to formulate your diagnosis? Choose up to five. 1. Diffuse muscle cramps 2. Alcohol intake 3. Effect of NSAIDs 4. Family history of back problems 5. Previous history of back injury 6. Shoulder pain 7. Weakness in right leg 8. Allergies 9. Knee problems 10. Urinary incontinence 11. Skin rashes 12. Previous effective physiotherapy 13. Headaches 14. Nocturia

QUESTION 18 (CASE 7)
What questions would you ask on history to help determine the cause of the bleeding? Choose up to five. 1. Family history of inflammatory bowel disease 2. Hematemesis 3. Recent weight loss 4. Nausea and vomiting 5. Change in bowel habits 6. Abdominal pain 7. Fever/chills 8. Family history of colo-rectal cancer 9. Blood mixed with stool or coating the surface 10. Past history of hemorrhoids 11. Dietary history 12. Smoking history 13. Pain on defecation 14. Allergies

QUESTION 22 (CASE 9)
What elements of the physical exam would you focus on? Choose up to four. 1. Examination of skin 2. Range of motion of lumbar spine 3. Blood pressure 4. Inspection of muscle bulk in lower extremities 5. Sensory exam of arms 6. Peripheral pulses 7. Reflexes in lower extremities 8. Motor testing of legs 9. Motor testing of arms 10. Sensory exam of legs 11. Bowstring sign 12. Respiratory rate 13. Temperature 14. Straight leg raise test 15. Abdominal exam 16. Rectal exam 17. Pulse 18. Range of motion of knee 19. Auscultation of chest 20. Romberg test

CASE 8
You are called to the nursery to assess a 2 hour old term newborn with respiratory distress. The only information the nurse gives you over the phone is that the baby was born this morning, had good Apgars and arrived in the nursery pink on room air and in no distress.

QUESTION 19 (CASE 8)
What are the most common diagnoses that would be in your differential for this infant? List up to three. 1. _____________________________________________________________ 2. _____________________________________________________________ 3. _____________________________________________________________

MCCQE 2002 Review Notes

Sample Questions 55

KEY FEATURE SCORING GUIDE


QUESTION 1, CASE 1
1. 2. 3. 4. 5. Maximum Number to be selected: The Number you selected (S): Is S greater than the Maximum? There are 5 correct answers: Number of correct answers you selected: YES NO = = 5 ___

QUESTION 5, CASE 2
1. 2. 3. 4. 5. Maximum Number to be selected: The Number you selected (S): Is S greater than the Maximum? There are 4 correct answers: Number of correct answers you selected: YES NO = = 4 ___

Your Score = 0. Continue. 2, 4, 8, 13, 20 = ___

Your Score = 0. Continue. 6, 13, 16, 19 = ___

QUESTION 2, CASE 1
1. 2. 3. 4. 5. Maximum Number to be selected: The Number you selected (S): Is S greater than the Maximum? There are 5 correct answers: Number of correct answers you selected: YES NO = = 5 ___

QUESTION 6, CASE 3
1. 2. 3. 4. Maximum Number to be listed: The Number you listed (L): Is L greater than the Maximum? There are 2 correct answers: 1. Delirium 2. Acute Confusional State 5. Number of correct answers you listed: = ___ YES NO = = 2 ___

Your Score = 0. Continue. 9, 13, 14, 18, 20 = ___

Your Score = 0. Continue.

QUESTION 3, CASE 2
1. 2. 3. 4. Maximum Number to be listed: The Number you listed (L): Is L greater than the Maximum? There are 8 correct answers: 1. 2. 3. 4. 5. 6. 7. 8. 5. Prematurity Difficult child History of parental child abuse History of substance abuse Social isolation Developmental delay Poverty Parental character/personality disorder = ___ YES NO = = 4

QUESTION 7, CASE 3
___ 1. Your Score = 0. Continue. 2. 3. 4. 5. Maximum Number to be selected: The Number you selected (S): Is S greater than the Maximum? There are 5 correct answers: Number of correct answers you selected: YES NO = = 4 ___

Your Score = 0. Continue. 1, 5, 12, 14, 15 = ___

QUESTION 8, CASE 3
1. 2. Maximum Number to be selected: The Number you selected (S): Is S greater than the Maximum? There are 5 correct answers: Number of correct answers you selected: YES NO = = 4 ___

Number of correct answers you listed:

QUESTION 4, CASE 2
1. 2. 3. 4. 5. Maximum Number to be selected: The Number you selected (S): Is S greater than the Maximum? There are 4 correct answers: Number of correct answers you selected: YES NO = = 4

3. 4. ___ 5. Your Score = 0. Continue. 7, 9, 14, 19 = ___

Your Score = 0. Continue. 3, 5, 6, 11, 19 = ___

56 Sample Questions

MCCQE 2002 Review Notes

KEY FEATURE SCORING GUIDE


QUESTION 9, CASE 4
1. 2. 3. 4. 5. Maximum Number to be selected: The Number you selected (S): Is S greater than the Maximum? There are 4 correct answers: Number of correct answers you selected: YES NO = =

. . . CONT.

QUESTION 13, CASE 5


4 ___ 1. 2. 3. 4. ___ 5. Number of correct answers you selected: = ___ Maximum Number to be selected: The Number you selected (S): Is S greater than the Maximum? There are 8 correct answers: YES NO = = 7 ___

Your Score = 0. Continue. 1, 4, 6, 8 =

Your Score = 0. Continue. 2, 3, 5, 6, 7, 12, 13, 15

QUESTION 10, CASE 4 QUESTION 14, CASE 5


1. 2. 3. 4. 5. Maximum Number to be selected: The Number you selected (S): Is S greater than the Maximum? There are 8 correct answers: Number of correct answers you selected: YES NO = = 7 1. ___ 2. Your Score = 0. Continue. 1, 2, 4, 5, 7, 8, 10, 14 = ___ 3. 4. 5. The Number you selected (S): Is S greater than the Maximum? There are 4 correct answers: Number of correct answers you listed: YES NO = ___ Maximum Number to be selected: = 4

Your Score = 0. Continue. 2, 6, 7, 10 = ___

QUESTION 11, CASE 4


1. 2. 3. 4. 5. Maximum Number to be selected: The Number you selected (S): Is S greater than the Maximum? There are 7 correct answers: Number of correct answers you selected: YES NO = = 6 ___

QUESTION 15, CASE 6


1. 2. 3. 4. Maximum Number to be listed: The Number you listed (L): Is L greater than the Maximum? There are 3 correct answers: 1. Biliary colic 2. Acute cholecystitis 3. Bile duct carcinoma 5. = = YES NO 3 Number of correct answers you listed: = ___ YES NO = = 3 ___

Your Score = 0. Continue. 3, 5, 7, 8, 9, 13, 18 = ___

Your Score = 0. Continue.

QUESTION 12, CASE 5


1. 2. 3. 4. Maximum number to be selected The number you listed (L): Is L greater than the Maximum? There are 4 possible answers: 1. Bacterial vaginosis 2. Vaginal/vulvar candidiasis 3. Trichomonas vaginalis 4. Chemical vaginitis 5. Number of correct answers you listed: = ___

QUESTION 16, CASE 6


___ 1. Your Score = 0. Continue. 2. 3. 4. 5. Maximum Number to be selected: The Number you selected (S): Is S greater than the Maximum? There are 7 correct answers: Number of correct answers you selected: YES NO = = 7 ___

Your Score = 0. Continue. 1, 2, 3, 5, 6, 7, 8 = ___

MCCQE 2002 Review Notes

Sample Questions 57

KEY FEATURE SCORING GUIDE


QUESTION 17, CASE 7
1. 2. 3. 4. Maximum Number to be listed: The Number you listed (L): Is L greater than the Maximum? There are 4 correct answers: 1. 2. 3. 4. 5. Hemorrhoids Anal fissure Colon or rectal cancer Proctitis or IBD or colitis = YES NO = =

. . . CONT.

QUESTION 20, CASE 8


3 ___ 1. 2. 3. 4. 5. Maximum Number to be selected: The Number you selected (S): Is S greater than the Maximum? There are 4 correct answers: Number of correct answers you selected: YES NO = = 4 ___

Your Score = 0. Continue.

Your Score = 0. Continue. 1, 6, 8, 9 = ___

QUESTION 21, CASE 9


Number of correct answers you listed: ___ 1. Maximum Number to be selected: The Number you selected (S): Is S greater than the Maximum? Essential answers are: Did you select both essential answers? NO YES There are 5 correct answers: Number of correct answers you listed: YES NO = = 5 ___ 2. 3. 1. 2. 3. 4. 5. Maximum Number to be selected: The Number you selected (S): Is S greater than the Maximum? There are 6 correct answers: Number of correct answers you selected: YES NO = = 5 ___ 4. 5.

QUESTION 18, CASE 7

Your Score = 0. Continue. 7 and 10 Your Score = 0. Continue. 3, 5, 7, 10, 12 = ___

Your Score = 0. Continue. 1, 5, 8, 9, 10, 13

6. = ___ 7.

QUESTION 19, CASE 8 QUESTION 22, CASE 9


1. 2. 3. 4. Maximum Number to be listed: The Number you listed (L): Is L greater than the Maximum? There are 4 correct answers: 4. 1. 2. 3. 4. 5. Transient tachypnea of the newborn or wet lung Meconium aspiration Pneumonia Cardiac disease = ___ 5. There are 8 correct answers: Number of correct answers you selected: 2, 4, 7, 8, 10, 11, 14, 16 = ___ YES NO = = 3 1. ___ 2. Your Score = 0. Continue. 3. The Number you selected (S): Is S greater than the Maximum? YES NO = ___ Your Score = 0. Continue. Maximum Number to be selected: = 6

Number of correct answers you listed:

58 Sample Questions

MCCQE 2002 Review Notes

You might also like